You are on page 1of 82

ForumIAS

Mains Marathon
Compilation
August, 2018
MAINS MARATHON COMPILATION August, 2018

Questions

General Studies – 1
Q. 1) Persuasion is a powerful force in daily life and has a major influence on society and a whole.
Comment. (GS-1)

Q.2)How did the early peasant rebellions differ from the later peasant rebellions in India’s Struggle for
freedom? Examine. GS 1

Q.3) Discuss the reasons why India is lagging behind in promotion of sports and culture? Highlight the
significance of progress in these fields for a country. (GS-1)

Q.4) India’s rapid urbanization has been proceeding apace for decades, but current slum policies have
been less than sufficient to deal with the socio-economic distress brewing in slums. Explain. (GS-1)

Q.5) After independence communalism was contained and weakened but not eliminated, for conditions
were still favorable for its growth. Explain (GS-1)

Q.6) The language problem was the most divisive issue in the first twenty years of independent India,
and it created the apprehension among many that the political and cultural unity of the country was in
danger. Discuss. (GS-1)

Q.7) A number of factors prompted the United States to enter the First World War Discuss. Also,
enumerate the various consequences of the United States entering the war? (GS-1)

Q.8) What were the various causes of French Revolution? Discuss the impacts this revolution had on
Europe. (GS-1)

Q.9) What was the Cuban missile crisis and what were the reasons behind it? What were its major
consequences? (GS- 1)

Q.10) During elections of 1930s Nazi party came out to become single largest party. What were the factors
that contributed in its popularity? (GS-1)

Q.11) Explain why racism became a more important part of Italian fascism in the 1930s. (GS-1)

General Studies – 2
Q.1) ‘Given the procedural and logistical challenges that holding of simultaneous elections pose, it would
be far more productive for political parties to focus on basic electoral reforms and find ways to curb
excessive election expenditure’. Discuss. (GS-2)

Q.2) Why there is an urgent need to revise India’s Bhutan policy? In your opinion what policy
alternatives does India can adopt to keep the relations with Bhutan cordial? (GS-2)
Created with by ForumIAS.com – The Knowledge Network for Civil Services. Visit http//forumias.com New!
http://forumiasacademy – Prelims & Mains Test Series

ForumIAS
MAINS MARATHON COMPILATION August, 2018

Q.3) National Health Protection Mission provide an overarching framework for health protection but
need an infrastructure upgrade to function properly. Explain. GS-2

Q.4) The rule of equality before law is not absolute and there are constitutional and other exceptions to it.
Discuss and also highlight the differences between ‘equality before law’ and ‘equal protection of laws’.
(GS-2)

Q.5) Why the article 32 has been called as ‘an Article without which constitution would be a nullity?
Discuss how writ jurisdiction of the Supreme Court differs from that of a high court. (GS-2)

Q.6) Highlight the reasons behind failure of online companies to curb circulation of online videos of
sexual violence against women and children. In your opinion what measures should government take to
deal with this menace? (GS-2)

Q.7) Supreme Court is hearing a plea challenging constitutional validity of adultery law. Critically
examine the reasons for which adultery law is being challenged. (GS-2)

Q.8) Human trafficking can no longer be understood only as a matter of crime as the newly introduced
Trafficking of Persons (Prevention, Protection and Rehabilitation) Bill, 2018 on the issue does, it is equally
a development issue. Explain. (GS-2)

Q. 9) What is political attitude? Discuss the factors that shape political attitude and how media influences
political attitude of public? GS-2

Q.10) Discuss the reasons that Prohibition of Employment of Manual Scavengers and Their Rehabilitation
Act 2013 came into being about 25 years ago but manual scavenging continues to be rampant in India.
Suggest some measures for the same.(GS-2)

Q.11) What were the reasons for which Fundamental Duties have repeatedly been called ineffective?
Highlight the need and importance of incorporating FDs in Indian Constitution. (GS-2)

Q. 12) The Punjab Cabinet’s decision to amend the law to make acts of sacrilege against the holy books of
major religions punishable with life imprisonment is retrograde and fraught with undesirable
consequences. Explain. (GS-2)

Q.13) Explain what are the individual responsibilities and collective responsibilities with reference to
Council of Ministers? Also highlight the differences between ‘council of ministers’ and ‘cabinet’? (GS-2)

Q.14) The Constitution itself declares that the Directive Principles of State Policy (DPSP) are fundamental
to the governance of the country. In the light of the statement, discuss the importance of DPSP and
highlight how these are different from the Fundamental Rights? (GS-2)

Q.15) RCEP ( Regional Comprehensive Economic Partnership) is not just beneficial for its member
countries, but also for the whole of Asia. Analyze. Also, discuss India’s concerns with RCEP. (GS-2)

Q.16) Malnutrition has emerged as a major challenge in India’s growth story. Discuss the challenges in
eradicating the problem, enumerating the governmental interventions in this regard. (GS-2)
Created with by ForumIAS.com – The Knowledge Network for Civil Services. Visit http//forumias.com New!
http://forumiasacademy – Prelims & Mains Test Series

ForumIAS
MAINS MARATHON COMPILATION August, 2018

Q.17) Do you think adultery law in India is in contravention to the Constitutional Provisions under Art 14
and 15. Critically analyze, with special emphasis on various governmental reports and committees’ views
on the issue. (GS-2)

Q.18) Discuss the effectiveness of Maintenance & welfare of parents’ senior citizen Act, 2007 in dealing
with problems faced by elderlies in India.(GS-2)

Q.19) The lateral entry scheme, if implemented properly, may foster more competitive spirit, break the
complacency of the higher civil servants and eventually prove to be a pioneering initiative in public
interest. Elucidate. (GS-2)

Q.20) There needs to be a road map to rescue private Industrial Training Institutes from their weak state.
Explain. Also suggest measures for skills development in India. (GS-2)

Q.21) Highlight the need for Universal Social Security Scheme in India. Discuss the major government
interventions for old ages in India. (GS-2)

Q.22) Street Vendors Act (Protection of Livelihood and Regulation of Street Vending), 2014 aimed at
changing the status quo of street vendor but the implementation gap are defeating the very purpose of
the Act. Discuss. (GS-2)

Q.23) Discuss the factors behind poor health of the private Industrial Training Institutes of India? In your
opinion what measures should be taken to improve their quality? (GS-2)

Q.24) World has changed and multilateral institutions now have to embed these changes. In the light of
the statement, highlight the importance of WTO in the present world order and needs to reform it. (GS- 2)

Q. 25) Highlight the issue faced by children at shelter homes for homeless children or those in conflict
with law. Discuss the provisions of Juvenile Justice (Care and Protection of Children) Act, 2015 that
governs these shelters and suggest some measures to deal with the issues faced by children. (GS-2)

Q.26) Discuss the controversial issues around article 35A of the Indian constitution. Should Supreme
Court repeal this provision? Provide valid arguments in support of your opinion. (GS-2)

Q.27) India’s law governing motor vehicles and transport is archaic, lacking the provisions necessary to
manage fast motorisation. Critically analyze the provisions of Motor Vehicles (Amendment) Bill. (GS-2)

Q.28) Tinkering with the RTE Act without sufficient thought will erode a major constitutional
achievement. Discuss.(GS-2)

Q.29) What is meant by social constraints? Are constraints of any kind necessary for enjoying freedom?
What is the difference between the negative and positive conception of liberty? (GS-2)

Q.30) Union Cabinet has approved an amendment to The Scheduled Castes and the Scheduled Tribes
(Prevention of Atrocities) Act to undo ‘dilution’ of the law by the Supreme Court. In the light of the
statement discuss the changes made by Supreme Court to SC/ST act along with the reasons given by it
and the recent changes proposed by government to it. (GS-2)

Created with by ForumIAS.com – The Knowledge Network for Civil Services. Visit http//forumias.com New!
http://forumiasacademy – Prelims & Mains Test Series

ForumIAS
MAINS MARATHON COMPILATION August, 2018

Q.31) Health care in India is obviously not egalitarian and becoming increasingly dependent on the
private sector to deliver health care. Discuss and suggest measures for equitable medical care. (GS-2)

Q.32) ‘BRICS has grown in influence in its first decade but is still far from achieving its initial goals’.
Discuss (GS-2)

Q. 33) Blasphemy law is not required in a secular state like India. Critically examine, with special
emphasis on SC judgements in this regard. (GS-2)

Q.34) Discuss the recent controversy around collegium system of Supreme Court. What is the criterion to
decide seniority in the Supreme Court. Is there there any need to bring changes to this criterion? (GS-2)

Q.35) India and Pakistan will resume their talks on various aspects of the Indus Waters Treaty. Discuss
the provisions of Indus water treaty and highlight the point of contentions between India and Pakistan
over it. (GS-2)

Q.36) Provide an account of the evolution of Draft National Register of Citizens and discuss the
controversies and challenges surrounding it? (GS-2)

General Studies – 3
Q.1) Discuss the factors behind India’s ailing indigenous defense manufacturing. How new Strategic
Partnership guidelines would be able to transform defense manufacturing in India? (GS-3)

Q. 2) Power loom sector, which contributes to 40 per cent of the nation’s man-made fabric demand, is in a
shutdown mode in Surat, discuss the challenges faced by the sector and steps taken by government to
deal with them. GS-3

Q.3) Enumerate the land-use categories as maintained in the Land Revenue Records and highlight the
patterns of land use changes that have taken place in last few decades in India. (GS-3)

Q.4) Critically analyse the government’s decision to turn down the foreign assistance for disaster relief
with special reference in case of Kerala. (GS-3)

Q.5) What is GST? What are the challenges in its implementation? What are the possible solutions? GS 3

Q.6) State the advantages and disadvantages of using Coal bed methane as a fuel in India. What is the
present scenario of CBM resources in India? GS-3

Q.7) Examine the issue of man-animal conflict in India. Also, evaluate the government’s strategy to
resolve the problem. (GS-3)

Q.8) Wind energy is India’s biggest source of renewable electricity by far, what are the factors
contributing to this achievement? Discuss the hurdles that wind energy sector is facing in faster
development of energy generation? (GS-3)

Created with by ForumIAS.com – The Knowledge Network for Civil Services. Visit http//forumias.com New!
http://forumiasacademy – Prelims & Mains Test Series

ForumIAS
MAINS MARATHON COMPILATION August, 2018

Q.9) Why does equality matters? How do we distinguish between political, economic and social
dimensions of equality? (GS-3)

Q.10) States need to reconsider their stand against Kasturirangan committee report on Western ghat in
the wake of recent calamity. Comment (GS-3)

Q.11) Discuss the roles that telemedicine can play in improving the health conditions of rural India? What
are the challenges in its implementing? (GS-3)

Q.12) Discuss the concept of ‘hothouse earth’. What measures are required to prevent a ‘hothouse earth’
pathway? (GS-3)

Q.13) TRAI has recently recommended the auction of spectrum for offering 5G services. How is the 5g
technology different to 4g technology and what would be its possible effects on global industries? (GS-3)

Q.14) A clampdown on the sale of oxytocin will have severe public health consequences. Critically
analyze. (GS-3)

Q.15) What are cryptocurrencies? Do you think that cryptocurrencies should be banned? (GS-3)

Q.16) Discuss the impact of GST in infrastructure sector. Suggest measures to be taken by the government
to boost growth in infrastructure sector in India. (GS-3)

Q.17) Examine the need for increasing public investment to boost growth in India. Also, discuss the
problems associated with public investment in India (GS-3)

Q.18) While safeguard duty on solar panels from China, Malaysia may benefit domestic manufacturers, it
will do a lot of damage to existing solar projects. Explain. And discuss the other factors that are hurting
the development of solar power in India. (GS-3)

Q.19) DNA Technology (Use and Application) Regulation Bill, 2018 can be an important tool in solving
crimes, but it is important that there are safeguards to protect human rights and prevent miscarriages of
justice. Discuss. (GS-3)

Q.20) Recently ministry of corporate affairs (MCA) has deregistered about 50,000 companies. Highlight
the threats posed by illegal shell companies and steps taken by government to deal with them. (GS-3)

Q.21) Instances of people losing their hard-earned money to Ponzi schemes keep coming to light. What
are Ponzi schemes? How these schemes run unhindered in India? In your opinion, what steps should be
taken to curb the menace of these schemes? (GS-3)

Q.22) Invasive species have contributed to 40% of all animal extinctions since the 17th century, discuss
the implications of the invasive species on the local environment and economy. What international
mechanisms are available to deal with them? (GS-3)

Q.23) Critically analyze the reasons for which union government has recently banned the import of
oxytocin and also discuss the legal provisions governing drugs ban in India. (GS-3)

Created with by ForumIAS.com – The Knowledge Network for Civil Services. Visit http//forumias.com New!
http://forumiasacademy – Prelims & Mains Test Series

ForumIAS
MAINS MARATHON COMPILATION August, 2018

Q.24) What are the reasons of urban flooding in India? Discuss the mechanism available to deal with
urban flooding and why it has proved to be ineffective in times of need? GS-3

Q.25) What is Biofuel. Discuss its applications, benefits and challenges in its application. (GS-3)

Q.26) Discuss the causes and implications of falling Rupee value at present in India. GS-3

Q.27) What are the causes behind disputes over land acquisitions in India. Would Land acquisition
(second amendment) bill prove to be effective in dealing with these disputes? provide valid arguments in
favour of your views(GS-3)

Q.28) Highlight the need for National security reforms and restructuring in India and discuss the steps
taken by government towards this goal. GS-3

Q.29) Reliable Internet connectivity has the potential to bridge the rural-urban economic divide and
revolutionise healthcare and education. Explain. (GS-3)

Q.30) Critically assess the need of military reform in India, giving special importance to
recommendations of Naresh Chandra Committee. (GS-3)

Q.31) Discuss the factors behind India’s ailing indigenous defense manufacturing. How new Strategic
Partnership guidelines would be able to transform defense manufacturing in India? (GS-3)

General Studies – 4
Q.1) Is morality important to lead a happy life? Support your answer with examples. GS 4

Q.2) “Emotional intelligence is your ability to recognize and understand emotions in yourself and others,
and your ability to use this awareness to manage your behavior and relationships.: Analyze. (GS4)

Q.3) Citizen’s Charter helps maintaining a healthy work culture. Do you agree? Explain with Examples.
(GS-4)

Q.4) Differentiate between dilemma and ethical dilemma with suitable examples. How does conscience
act as a source of ethical guidance? (GS-4)

Q.5) What do you understand by corporate social responsibility? What relationship does corporate
governance share with corporate social responsibility? (GS-4)

Q.6) What do you understand by “entrepreneurial governments”? Do you think civil servants need to
develop entrepreneurial behavior? (GS-4)

Created with by ForumIAS.com – The Knowledge Network for Civil Services. Visit http//forumias.com New!
http://forumiasacademy – Prelims & Mains Test Series

ForumIAS
MAINS MARATHON COMPILATION August, 2018

Answers

General Studies – 1

Q. 1) Persuasion is a powerful force in daily life and has a major influence on society and a whole.
Comment. (GS-1)

Answer:

Persuasion is a process aimed at changing a person's or a group's attitude or behavior toward some event
or person by using written, spoken words or visual tools to convey information, feelings, or reasoning, or
a combination thereof.
It is a tool in the pursuit of personal gain, such as election campaigning, giving a sales pitch, or in trial
advocacy. Using one's personal or positional resources to change people's behaviors or attitudes.

Persuasion in present daily life & influence society:


1. Politics, legal decisions, mass media, news, and advertising are all influenced by the power of
persuasion and influence us in turn.
2. Public service campaigns that urge people to recycle or quit smoking are great examples of
persuasion used to improve people’s lives.
3. The number of persuasive messages has grown tremendously Ex: we may come across hundreds
of advertisements on a daily basis
4. Persuasive communication travels far more rapidly. Television, Radio and the Internet all help
spread persuasive messages very quickly.
5. Contemporary persuasion is much more subtle. Businesses sometimes carefully craft very
specific image designed to urge viewers to buy products or services in order to attain that
projected lifestyle.
6. Persuasion is more complex as consumers are more diverse and have more choices, so marketers
have to be savvier when it comes to selecting their persuasive medium and message.

Q.2)How did the early peasant rebellions differ from the later peasant rebellions in India’s Struggle for
freedom? Examine. GS 1

Early Peasant rebellions:


• After 1857’s revolt, The British had crushed down native princes and zamindars. Hence farmers
themselves became main force of agitations.
• Target was sometimes government, sometimes moneylender, sometimes landlord/ zamindar
• Territorial reach was limited and was not organized on mass-scale.
• The localised nature of these revolts were seen in Moplah uprising was due to hike in revenue
demands and reduction of field sizes
• Often spontaneous with limited coordination
• lacked continuity or long term struggle.
• Never threatened British supremacy
Created with by ForumIAS.com – The Knowledge Network for Civil Services. Visit http//forumias.com New!
http://forumiasacademy – Prelims & Mains Test Series

ForumIAS
MAINS MARATHON COMPILATION August, 2018

• Farmers didn’t mind paying rent, revenue, interest on debt but only agitated when they were
raised to an abnormal level.
• Lacked understanding of colonial economic system or divide and rule policy of the British.
Farmers’ agitations were based within framework of old social order, hence often failed because
government could woo a faction by granting them concession and hence movement would
collapse.
• Government issued a notification that the Indian farmers cannot be compelled to grow indigo
and that it would ensure that all disputes were settled by legal means. By the end of 1860, Indigo
planters should down their factories and cultivation of indigo was virtually wiped out from
Bengal.
• In the deccan riots Initially government resorted to use of police force and arrest. but later
appointed a commission, passed Agriculturists Relief Act in 1879 and on the operation of Civil
Procedure Code.Now the peasants could not be arrested and sent to jail if they failed to pay their
debts

Later peasant rebellions:

• Earlier kisan movements usually didn’t demand abolition of Zamindari. They merely wanted a
fair system of land revenue and land tenancy. But these new movements strongly demanded for
abolition of Zamindari.
• Even when they were unsuccessful, they created a climate which necessitated the post-
independence land reforms and abolition of Zamindari.
• Earlier movements were by and large non-violent. But now they turned militant e.g. Telangana
movement in Hyderabad state and the Tebhaga movement in Bengal. Similarly All India Kisan
Sabha openly preached militancy, violence against Zamindars.
• Peasant leaders anticipated freedom and new social order. Hence new movements started with
renewed vigour especially after WWII.
• The circumstances and the awareness of the nationalists and the people has led to the goals of
peasant revolts being different in the 19th and the 20th century.

Q.3) Discuss the reasons why India is lagging behind in promotion of sports and culture? Highlight the
significance of progress in these fields for a country. (GS-1)

Answer:

India has the 2nd largest population in the world. Our performance in various sporting events has been

Reasons for lagging performance:


1. Nature of school education is mostly monotonous and does not encourage creativity in children.
The focus of education has remained to be more result oriented. Our education system has
reduced the notions of competence and merit only to that of science, thereby denying the
greatness inherent in so many other domains.
2. State of administration – institutions are not held accountable to the results of that funding.

Created with by ForumIAS.com – The Knowledge Network for Civil Services. Visit http//forumias.com New!
http://forumiasacademy – Prelims & Mains Test Series

ForumIAS
MAINS MARATHON COMPILATION August, 2018

3. Cultural response to the idea of excellence – it is difficult for a person to stand out in terms of
high quality work since the system has little support for excellence.
4. Countless artists and musicians struggle to survive in spite of creating great work. There is no
monthly salary, provident fund and pension for some of the greatest artists, performers, writers
and others. This lack of state support to creativity and talent is a reason for our performance.

Significance for a country:

1. It enhances the status of a country on the world platform. It can be tapped to enhance the soft
power of the country.
2. It provides multiple opportunities for individuals to excel in different fields.
3. The advances in attitude reflects in the state of development in a country. Britain embraced
scientific revolution before it advanced to industrial revolution and became a large power.
4. It effectively taps the demographic dividend of our country.

Q.4) India’s rapid urbanization has been proceeding apace for decades, but current slum policies have
been less than sufficient to deal with the socio-economic distress brewing in slums. Explain. (GS-1)

Answer: According to the U.N. World Urbanization Prospects 2018 report, about 34% of India’s
population now lives in urban areas. This is an increase of about 3% since the 2011 Census. It is said that
by 2030, 50% of India’s population would reside in urban areas.

Most cases of migration are out of rural distress. These people end up in the urban slums where the
infrastructure and quality of life are very poor.

Deficiency of slum policies:


1. There are no concrete figures on the temporary settlements as slums have a fluid definition. 2011
Census estimated 65mn people in slums much less than the UN-HABITAT’s 2014 estimate of
104mn.
2. Current slum policies primarily focus on housing, relocation or in-situ development of multi-
storey complexes. But they miss the socio-economic distress in slums. A survey by researchers
from Netherlands, U.S. and a local NGO revealed that over 70% of families in slums live in debt.
3. Other financial problems in slums are ignored. As there is no access to formal financial systems,
any borrowing comes from private money lenders at high interest rates.
4. Lack of infrastructure development in these areas is a problem. Water and electricity are
disproportionately more expensive here as they are forced to rely on the grey market rather than
on formal channels.
5. Lack of formal identification of migrants in slums deprives them of the social welfare benefits
they used to receive back in the villages.
Thus there is a need for a slum policy which does not impose a one-size-fits-all approach.

Created with by ForumIAS.com – The Knowledge Network for Civil Services. Visit http//forumias.com New!
http://forumiasacademy – Prelims & Mains Test Series

ForumIAS
MAINS MARATHON COMPILATION August, 2018

Q.5) After independence communalism was contained and weakened but not eliminated, for conditions
were still favorable for its growth. Explain (GS-1)

Answer: It is a belief that people who follow the same religion have common secular interests i.e. they
have same political, economic and social interests.

How it was contained after independence:


1. The constitution of India provided a framework for equality, justice to all religions.
2. The country was ruled on democracy which granted equal political rights to individuals from all
communities to contest elections and to hold public office.
3. Minority communities were given fundamental rights under Article 28,29 and 30, according to
which they are free to manage their educational institutions and have right to conserve their own
culture.
4. Since Independence, India has been pursuing the ideal of nation-building based on secularism.

Why it was not eliminated:


1. In the absence of Uniform civil code, there is a perception that all communities have divergent
and contradictory interests.
2. Community based pressure groups are bargaining for their respective communities life AIMPLB
against Triple Talaq. These communities compete for power and resources.
3. Blind belief in one’s religious principles and lack of tolerance towards others’ religious principles
makes religion a source of conflict.
4. Politics is also a source because for the sake of vote bank, the policy of appeasement, selection of
candidates on the basis of community, sect, sub-sect and caste, and flaring up religious
sentiments at the time of elections led to the rise of communalism.
5. Socio economic conditions of India are still unequal. Many challenges like poverty, illiteracy and
unemployment confront Indian society and are becoming threat for its diversity.
6. External elements (including non-state actors) also have a role in worsening the problem of
communalism.
This communal violence was witnessed in the form of Anti-Sikh riots of 1984, Babri Masjid destruction
and Godhra violence.

Q.6) The language problem was the most divisive issue in the first twenty years of independent India,
and it created the apprehension among many that the political and cultural unity of the country was in
danger. Discuss. (GS-1)

Answer: Language acted as a divisive issue in India with over the Official Language Act and the
linguistic reorganization of states.

Official Language Act:


1. The debate started around which language the central govt should use to carry on work and
maintain contact with the states. Centre proposed Hindi as the official language of the country.

Created with by ForumIAS.com – The Knowledge Network for Civil Services. Visit http//forumias.com New!
http://forumiasacademy – Prelims & Mains Test Series

ForumIAS
MAINS MARATHON COMPILATION August, 2018

2. This became a controversy as it took the form of opposition to Hindi language and created a
conflict between the Hindi speaking and non-Hindi speaking regions of the country.
3. The debate was also around the timeframe for switching to Hindi as the official language.
4. But protests arouse in Southern states opposing Hindi as the spread of the language didn’t
happen as fast as expected.
5. Thus the idea of imposing Hindi as the official language was dropped and the use of English
continued. Steps are being taken from time to time to increase the usage of Hindi language.

Linguistic reorganisation of States:


1. Reorganisation of states based on language was proposed by early congress leaders and it was
demanded by states after independence.
2. Case for linguistic states as administrative units was based on similar language, customs. It was
thought to be feasible for democracy and administration.
3. It was challenged by successive commissions like S K Dar commission which quoted that
administrative convenience needs to be given priority.
4. After successive demands by states, the State Reorganisation Commission recommended the
reorganisation of states on linguistic basis.
5. Thus the states of Maharashtra, Gujarat, punjab, Haryana, Andhra Pradesh were formed based
on language basis.
However, the language problem proved that Indian democracy could withstand diversity by
accommodating the needs of people.

Q.7) A number of factors prompted the United States to enter the First World War Discuss. Also,
enumerate the various consequences of the United States entering the war? (GS-1)

Answer: US entered the first world war after the sinking of the British ocean liner Lusitania by a German
U-boat in 1915 where 128 Americans died. Effectively, it stayed neutral from 1914 to 1917.

Factors which prompted US entry:


1. The invasion of neutral Belgium and stories of German atrocities in the country which shocked
and outraged the Americans. Stories of unarmed civilians being killed and small towns being
destroyed circulated throughout the press.
2. Trade between the Central Powers and U.S. was severely curtailed by Britain’s naval blockade of
Germany.
3. U.S. banks also provided the warring nations with loans, the bulk of which went to the Allies.
4. A German submarine sank the British ocean liner Lusitania, resulting in the deaths of nearly
1,200 people, including 128 Americans. This strained diplomatic relations between US and
Germany. It also turned public opinion against Germany. Despite promises, German U-boats
sank a series of U.S. merchant ships, resulting in multiple casualties.
5. Zimmerman telegram proposed an alliance between Germany and Mexico that if America joined
the war on the side of the Allies. They promised that Germans would support the Mexicans in
regaining the territory they’d lost in the Mexican-American War.
6. After this incident, US officially entered the First World War.
Created with by ForumIAS.com – The Knowledge Network for Civil Services. Visit http//forumias.com New!
http://forumiasacademy – Prelims & Mains Test Series

ForumIAS
MAINS MARATHON COMPILATION August, 2018

Consequences of US entry:
1. It brought the war on European front to an end.
2. US explanded its trade heavily taking advantage of Europe’s preoccupation with war. Thus it
could rise as an economic power by the end of the war. The balance of economic power began the
transfer from the drained nations of Europe to America.
3. U.S’ military was turned into a large-scale fighting force with the intense experience of modern
war

Q.8) What were the various causes of French Revolution? Discuss the impacts this revolution had on
Europe. (GS-1)

Answer: French Revolution is the revolutionary movement that shook France between 1787 and 1799. It
denotes the end of the ancien régime in France.

Causes of French Revolution:


Social causes:
1. French society was divided into classes, or estates. There were two privileged classes, the clergy
and the nobles. In a population of 25,000,000 people, these two classes together owned about 40%
of the total land of France. The life of the nobility was everywhere characterized by extravagance
and luxury.
2. The rest of the people of France were called the Third Estate. They were the common people and
numbered about 95%of the total population. Most of them lived in poverty and had to pay a
large share of taxes disproportionate to their incomes.

The Intellectual Movement:


1. The French revolutionary philosophers asserted that man was born to be happy. They believed
that man can attain happiness if reason is allowed to destroy prejudice and reform man’s
institutions.
2. The clergy were the first to feel the brunt of the French philosophers. A long series of scientific
advances dating from the Renaissance helped in their campaign against the clergy.
3. The French economists of the time were called ‘physiocrats’. They believed in “Laissez faire”
4. Montesquieu, thought about the kind of government that is best suited to man and outlined the
principles of constitutional monarchy.

Immediate causes:
1. Louis XVI’s need for money compelled him to agree to a meeting of the States General— the old
feudal assembly. Louis wanted to obtain its consent for new loans and taxes. All three Estates
were represented in it but each one held a separate meeting.
2. Members of the Third Estate, claiming to represent 96 per cent of the nation’s population,
declared themselves the National Assembly.
3. This led to the fall of Bastille and the revolution spread to rural parts of France. Soon, the king
was forced to flee.

Created with by ForumIAS.com – The Knowledge Network for Civil Services. Visit http//forumias.com New!
http://forumiasacademy – Prelims & Mains Test Series

ForumIAS
MAINS MARATHON COMPILATION August, 2018

Impact on Europe:
1. It inspired revolutionary movements in almost every country of Europe. Eg., the 1830 and 1848
revolutions.
2. Even though the old ruling dynasty of France had been restored to power in 1815, and the
autocratic governments of Europe found themselves safe for the time being, the rulers found it
increasingly difficult to rule the people.
3. Some of the changes that took place in many parts of Europe in the early 19th century were the
immediate, direct consequences of the Revolution and the Napoleonic wars.
4. The wars in which France was engaged with other European powers had resulted in the French
occupation of vast areas of Europe for some time.
5. The French soldiers, wherever they went, carried with them ideas of liberty and equality shaking
the old feudal order. They destroyed serfdom in areas which came under their occupation and
modernized the systems of administration.
6. Under Napoleon, the French had become conquerors instead of liberators. The countries which
organized popular resistance against the French occupation carried out reforms in their social
and political system. The leading powers of Europe did not succeed in restoring the old order
either in France or in the countries that the Revolution had reached.
7. The political and social systems of the 18th century had received a heavy blow. They were soon
to die in most of Europe under the impact of the revolutionary movements that sprang up
everywhere in Europe.

Q.9) What was the Cuban missile crisis and what were the reasons behind it? What were its major
consequences? (GS- 1)

Answer: Cuban missile crisis of 1962 is a major confrontation that brought the United States and Soviet
Union close to war over the presence of Soviet nuclear-armed missiles in Cuba.

Reasons behind it:


1. Prevailing Cold War between US and the USSR is the major factor.
2. As US placed missiles in Europe, USSR placed them in Cuba. The lack of trust on one another led
to fears over a potential conflict.
3. Fidel Castro took power in Cuba and nationalised American companies in Cuba. In retaliation,
the Americans stopped all aid to Cuba, and all imports of Cuban sugar. He looked up to USSR
for help and USSR signed an agreement to buy 1 million tonnes of Cuban sugar every year. This
fuel led tensions. After this, Castro favours Communism.
4. After the Bay of Pigs invasion, Russia publicly promised – weapons to defend Cuba against
America. This put America at a threat.
5. The arms race-caused fear and suspicion.
6. Creation of NATO-Berlin Blockade

Consequences:
1. Both sides were scared to escalate the crisis. This led to establishment of direct communications
and somewhat less aggressive behavior to avoid any future conflict.

Created with by ForumIAS.com – The Knowledge Network for Civil Services. Visit http//forumias.com New!
http://forumiasacademy – Prelims & Mains Test Series

ForumIAS
MAINS MARATHON COMPILATION August, 2018

2. The LIMITED TEST BAN TREATY: In August 1963, the USA and USSR agreed to ban the testing
of all nuclear weapons in space, in the sea and above ground. Underground tests were still
permitted.

3. The leaders of the Soviet Union were determined never again to be pushed around by America.
Therefore, the Soviet government made every effort to catch up in the Arms Race. By 1965,
America and the USSR were on equal footing in terms of their nuclear capability. This created
greater stability in the relationship between the two superpowers. American and Russian leaders
realised that any nuclear war was going to destroy both countries and therefore this was known
as the doctrine of Mutually Assured Destruction (MAD), giving both countries excellent reason to
avoid war.

4. France decided to leave the NATO. In the event of a nuclear war between the USA and USSR, the
members of the NATO would be obliged to fight alongside America. France feared that it would
be destroyed by this. In 1966, France ended its military alliance with America and began to
develop its own nuclear missiles.

Q.10) During elections of 1930s Nazi party came out to become single largest party. What were the factors
that contributed in its popularity? (GS-1)

Answer: In the 1932 elections, Nazis became the single largest party winning 230 seats out of 608.

Factors for their huge popularity are:


1. They claimed that Marxists, Jews are the reason for real troubles to Germany. They promised
prosperity to the nation by ridding Germans of them .
2. They tapped on the huge sentiment against Versailles treaty and promised to overthrow the
arrangement.
3. The organisational abilities of Hitler made the Nazi army popular among youth.
4. Fearing the communists, wealthy landowners and industrialists encouraged the Nazis’ policy.
5. They also tapped on the weaknesses of the govt of Weimar republic, which was inherently
weak, dull and failed to maintain law and order.
6. The economic crisis of 1929 helped Hitler in attaining the power. The widespread
unemployment gained mass support for Hitler.

Q.11) Explain why racism became a more important part of Italian fascism in the 1930s. (GS-1)

Answer: Racial policy is an important component of the Fascist state introduced in ITaly under
Mussolini. Though race did not make a component for a long time, it occupied a main place later.

It was not an important part of policy earlier:


1. Mussolini encouraged Zionism at one time as he thought it useful to embarrass British.
2. Many leading members of the Fascist party were Jews.

Created with by ForumIAS.com – The Knowledge Network for Civil Services. Visit http//forumias.com New!
http://forumiasacademy – Prelims & Mains Test Series

ForumIAS
MAINS MARATHON COMPILATION August, 2018

Why racism became important part of the policy?


1. Mussolini believed that Italians belonged to Aryan race that was superior. He was worried that
the slaves intermarried with Aryans and create a wrong impression of Italian character.
2. After Britain and France received Abyssinian invasion negatively, Italy was pushed to make an
alliance with Germany. He made the Rome-Berlin axis with Hitler in 1936.
3. The possession of territory in Africa needs to be justified in the name of superiority of race. The
Charter of Race published in 1938 claimed that Arabs, Africans and Jews were all inferior races.

Created with by ForumIAS.com – The Knowledge Network for Civil Services. Visit http//forumias.com New!
http://forumiasacademy – Prelims & Mains Test Series

ForumIAS
MAINS MARATHON COMPILATION August, 2018

General Studies – 2

Q.1) ‘Given the procedural and logistical challenges that holding of simultaneous elections pose, it would
be far more productive for political parties to focus on basic electoral reforms and find ways to curb
excessive election expenditure’. Discuss. (GS-2)

Guidelines Provide procedural and logistical challenges

Describe the need for basic electoral reforms and curbing excessive election expenditure and how that
would be more beneficial

Answer: The idea of holding elections simultaneously to Lok Sabha and State Legislative Assemblies is
being debated.

Procedural and logistic challenges associated with it:

Procedural:
1. There is the basic requirement of a legal framework under which the extension or curtailment of
the term of any Assembly is constitutionally permissible.
2. Altering the term of an Assembly needs an amendment to the Constitution.
3. Wider political consensus and legislative cooperation from various parties is required for holding
simultaneous elections.
Logistic:

1. Simultaneous elections demand a massive increase in the number of electronic voting machines
(EVMs) and voter-verifiable paper audit trail (VVPAT) units.
2. It requires deployment of large no of paramilitary forces at one time across the country.
3. More no of staff need to be deployed at once. Now the officers rotate on the duty at different
times in different states.

Need for electoral reforms:


1. To address criminalisation of politics.
2. To reduce role of black money in elections.
3. To bring in more honest people into politics and make politics citizen-centric.

Need to curb election expenditure:


1. High electoral spending distorts the level playing field between various political parties.
2. Most of the time, corporates donating to the parties gain back in the form of favourable policies
or corruption.
3. Vote buying by various political parties distorts the deepening of democracy.
As simultaneous elections compromise federalism and accountability at large, it is better to bring in
electoral reforms. The most important reasons cited for simultaneous elections are large cost of elections
and constant engagement in electoral campaigns. These can be addressed naturally if the larger electoral
reforms are put in place.
Created with by ForumIAS.com – The Knowledge Network for Civil Services. Visit http//forumias.com New!
http://forumiasacademy – Prelims & Mains Test Series

ForumIAS
MAINS MARATHON COMPILATION August, 2018

Q.2) Why there is an urgent need to revise India’s Bhutan policy? In your opinion what policy
alternatives does India can adopt to keep the relations with Bhutan cordial? (GS-2)

Answer: The bilateral relations between the Bhutan and India have been traditionally close and both
countries share a ‘special relationship’. India remains influential over Bhutan’s foreign policy, defence
and commerce.

Need to revise Bhutan’s policy:


1. Bhutan’s national debt is owed mostly to India for hydropower loans. The hydropower projects
where delays in constructing and commissioning in Bhutan by Indian companies led to the
country’s burgeoning national debt.
2. Need to respect the sovereignty, security and self-sufficiency of Bhutan.
3. Growing voices for Bhutan’s sovereignty after repeated Indian interferences in the affairs of the
country. The recent Doklam crisis and UPA’s decision in 2013 to cut cooking gas subsidy before
the 2013 elections in Bhutan has been shown as proof of Indian interference.
4. India’s power-surplus status and renewable energies like wind and solar power will make it
difficult for Bhutan to ensure that its hydropower sector becomes profitable. Otherwise, India
will be accused of “debt-trapping”.
5. The goods and services tax hurts Bhutanese exporters, and demonetisation has left scars on the
banking system.
6. Doklam has long been discussed as part of a “package solution” to Bhutan-China border dispute.
After the recent crisis, it could become a point of India-China dispute.

Policy alternatives:
1. India should keep its high-profile visits at an arms length from the election process in Bhutan.
2. To address debt, though the government agreed to raise tariffs for the original hydropower plant
in Chukha (by about 30 paisa per unit), other tariffs will need to be renegotiated too.
3. Focus on policing cross-border trade better.
4. Need to address the concerns of Bhutan in any border discussions including India, China and
Bhutan at Doklam.

Q.3) National Health Protection Mission provide an overarching framework for health protection but
need an infrastructure upgrade to function properly. Explain. GS-2

Answer: The NHPS will cover over 10 crore poor and vulnerable families and approximately 50 crore
beneficiaries. It provides coverage upto 5 lakh rupees per family per year for secondary and tertiary care
hospitalization.

It provides the framework for health protection:


1. Benefits of the scheme are portable across the country and a beneficiary covered under the
scheme will be allowed to take cashless benefits from any public/private empanelled hospitals
across the country.
Created with by ForumIAS.com – The Knowledge Network for Civil Services. Visit http//forumias.com New!
http://forumiasacademy – Prelims & Mains Test Series

ForumIAS
MAINS MARATHON COMPILATION August, 2018

2. It will be an entitlement based scheme and entitlement is decided on the basis of deprivation
criteria in the SECC database.
3. The beneficiaries can avail benefits in both public and empanelled private facilities.
4. In partnership with NITI Aayog, a robust and interoperable IT platform will be made operational
which will entail a paperless, cashless transaction.

Why it needs an infrastructure upgrade:


1. Majority of the families will be rural and the secondary and tertiary public hospital infrastructure
suffers from severe efficiency and accountability problems.
2. Though national schemes on health provide a framework, execution lies with State governments.
These state governments often do not have the necessary infrastructure in place to implement the
schemes.
3. There are problems with the distribution of hospitals, which are mostly concentrated in urban
areas and in few developed states.
4. The existing human resources are limited. To achieve a modest doctor-to-population ratio of
1:1,000, India will need 2.07 million more doctors by 2030.
5. Even the recommended funding of 2.5% GDP on public health is not met by successive
governments.
6. There is still a lack of regulation on the pricing of drugs and services provided by private
healthcare providers across the country.

Q.4) The rule of equality before law is not absolute and there are constitutional and other exceptions to it.
Discuss and also highlight the differences between ‘equality before law’ and ‘equal protection of laws’.
(GS-2)

Answer: Equality before law connotes: 1. the absence of any special privileges in favour of any person 2.
the equal subjection of all persons to the ordinary law of the land administered by ordinary law courts,
and 3. no person is above the law.

Exceptions to Equality before law:


1. The President of India and the Governor of States enjoy immunities under Art 361. These make
them immune from prosecution to civil or criminal proceedings during office.
2. No person shall be liable to any civil or criminal proceedings in any court in respect of the
publication in a newspaper of a substantially true report of any proceedings Legislature.
3. Article 105/194 guarantee that No member of Parliament(SL) shall be liable to any proceedings in
any court in respect of anything said or any vote given by him in Parliament or any committee
thereof.
4. Article 31-C provides that the laws made by the state for implementing the DPSP contained in
clause (b) or clause (c) of Article 39 cannot be challenged on the ground that they are violative of
Article 14. SC – “where Article 31-C comes in, Article 14 goes out”.
5. The foreign sovereigns, ambassadors and diplomats enjoy immunity from criminal and civil
proceedings. The UNO and its agencies enjoy the diplomatic immunity.

Created with by ForumIAS.com – The Knowledge Network for Civil Services. Visit http//forumias.com New!
http://forumiasacademy – Prelims & Mains Test Series

ForumIAS
MAINS MARATHON COMPILATION August, 2018

Differences between equality before law and equal protection of laws:


1. The concept of ‘equality before law’ is of British origin while the concept of ‘equal protection of
laws’ has been taken from the American Constitution.
2. Equal protection of laws connotes: 1. equality of treatment under equal circumstances 2. similar
application of same laws to all persons who are similarly situated, and 3. like should be treated
alike without any discrimination.
3. Equality before Law is a negative concept while the Equal protection of laws is a positive concept.

Q.5) Why the article 32 has been called as ‘an Article without which constitution would be a nullity?
Discuss how writ jurisdiction of the Supreme Court differs from that of a high court. (GS-2)

Answer: Art 32 provides the fundamental right to constitutional remedies. It provides the right to move
the Supreme Court or High Court by appropriate proceedings for the enforcement of the Fundamental
Rights is guaranteed.

Importance of Art.32: A mere declaration of fundamental rights in the Constitution is meaningless,


useless and worthless without providing an effective machinery for their enforcement, if and when they
are violated. Hence, Article 32 confers the right to remedies for the enforcement of the fundamental rights
of an aggrieved citizen. That is why Dr Ambedkar called Article 32 as the most important article of the
Constitution—‘an Article without which this constitution would be a nullity.

Differences in writ jurisdiction between SC and HC:

Created with by ForumIAS.com – The Knowledge Network for Civil Services. Visit http//forumias.com New!
http://forumiasacademy – Prelims & Mains Test Series

ForumIAS
MAINS MARATHON COMPILATION August, 2018

Q.6) Highlight the reasons behind failure of online companies to curb circulation of online videos of
sexual violence against women and children. In your opinion what measures should government take to
deal with this menace? (GS-2)

Answer: Reasons for failure:


1. The compliance of service providers to the previous guidelines in this regard is mediocre.
2. Issue of tracing the origin of “unlawful” content is a big challenge.
3. Assistance to law enforcement agencies on identification of offender is delayed and meta data of
end to end communication is not provided.
4. Govt instructed the companies to set up an easy reporting mechanism for public on their
platform. While Facebook has complied partially, Whatsapp and Twitter are yet to comply

Measures to be taken by government:


1. Bring directives to reduce the time taken by intermediary to comply with content removal
requests under Sections of the IT Act.
2. Request the companies to employ agencies for identification and removal of sexually violent
content.
3. The companies can be asked to deploy “proactive monitoring tools.”
4. Intermediaries should share certain data with law enforcement agencies to identify the origin of
such content.
5. Intermediaries should be asked to keep a complete trail of forwarding of unlawful content and
verify and maintain identifiers of the users that can help attribute information to the users.
6. Bring in data localisation measures like China to make the tracking of individuals easy.
7. Penalising the companies for lack of compliance to various directives.

Q.7) Supreme Court is hearing a plea challenging constitutional validity of adultery law. Critically
examine the reasons for which adultery law is being challenged. (GS-2)

Answer: Adultery refers to extra-marital voluntary sexual intercourse. Provisions in Section 497 of
Indian Penal Code mandates a maximum punishment of five years in prison for the man who had
consensual sexual intercourse with a married woman. But it does not punish the
married/unmarried/divorced/widowed woman who had consensual sexual intercourse with a married
man.

Why adultery law is challenged?


1. This is contested as patriarchal, parochial and paternalistic because it considers that women
could not be punished as they were vulnerable.
2. The section goes in contravention of Articles 14 and 15 of the Constitution of India. By punishing
only a man for adultery, the law is discriminating against citizens based on “sex”, covered under
Article 15.
3. It is also alleged that by not putting women in an equally culpable position as men, a licence of
sorts was being given to them to commit and abet crime.

Created with by ForumIAS.com – The Knowledge Network for Civil Services. Visit http//forumias.com New!
http://forumiasacademy – Prelims & Mains Test Series

ForumIAS
MAINS MARATHON COMPILATION August, 2018

4. According to Section 198 (1) of CrPC, only “aggrieved party” can bring forth a complaint on cases
of adultery. Section 198 (2) clarifies that only the husband of the erring wife is the “aggrieved
party” for purposes of Section 198 (1).
5. Decisional autonomy involves bodily and sexual autonomy. These are integral to right to privacy.
Restricting two consenting individuals from exercising this right runs counter to the Constitution.

The 42nd Law Commission of India Report and the Malimath Committee on Criminal Law Reforms
suggested that the section be amended to the effect that both the erring wife and the paramour be held
equally liable for adultery.

Q.8) Human trafficking can no longer be understood only as a matter of crime as the newly introduced
Trafficking of Persons (Prevention, Protection and Rehabilitation) Bill, 2018 on the issue does, it is equally
a development issue. Explain. (GS-2)

Answer: Human Trafficking is the trade of humans for the purpose of forced labour, sexual slavery, or
commercial sexual exploitation for the trafficker or others.

The Trafficking of Persons (Prevention, Protection and Rehabilitation) Bill, 2018 is being criticised on the
following issues.

Why it is not just another crime


1. The conviction rate, investigations, prosecutions and convictions in India are “disproportionately
low” relative to the scale of human trafficking.
2. Its approach to trafficking is based on a prosecution-driven, raid-rescue-rehabilitation model.
3. It ignores India’s own rich jurisprudence on labour exploitation and is a highly punitive
legislation.
4. It fails to respond to the precarious working conditions of millions of Indians.

Why is it a development issue


1. Men, women, and children in debt bondage – inherited from previous generations – are forced to
work in brick kilns, rice mills, embroidery factories and agriculture.
2. Most of India’s trafficking problem is internal and those from the most disadvantaged social
strata – lowest caste Dalits, members of tribal communities, religious minorities, and women and
girls from excluded groups – are most vulnerable.
3. Within India, some are subjected to forced labour in sectors such as construction, steel, garment
and textile industries, wire manufacturing for underground cables, biscuit factories, pickling,
floriculture, fish farms and ship breaking.
4. Traffickers use false promises of employment or arrange sham marriages within India or Gulf
states and subject women and girls to sex trafficking.

Created with by ForumIAS.com – The Knowledge Network for Civil Services. Visit http//forumias.com New!
http://forumiasacademy – Prelims & Mains Test Series

ForumIAS
MAINS MARATHON COMPILATION August, 2018

Q. 9) What is political attitude? Discuss the factors that shape political attitude and how media influences
political attitude of public? GS-2

Answer:

A political attitude is simply the way we think or feel about our government and related social and
economic issues. It defines our like or dislike for a political person, party or ideology.
Ex: A person with liberal political attitude supports basic ideas of liberty, equality and democracy but
want reforms in the system albeit not in extreme/violent manner but through constitutional / legal
means.
A person’s political attitude is shaped not by any single, but complex combination of factors

Factors that shape political attitude


1. Religion shapes a person’s moral attitude, moral attitude will predict political attitude. Ex:
people who believes a religion very seriously, may support a party or a person belonging to the
same religion
2. Age - In general older people will be more conservative. Young people will be more liberal /
radical.
3. Economic Status - Poor will align towards communist / socialist ideology. He’ll vote for a party
promising to get food, fertilizer and kerosene subsidy. Rich will align towards right wing- free
market, capitalism.
4. Residence - People belonging to a particular region will support a party that particularly
promises something to the region or people of that region. This can be observed in the states
where local parties dominate in assembly elections.
5. Family - Children more likely to espouse the political ideology of their parents.
6. Race - If a race feels they’re deprived of opportunities because of other races, then more likely to
lean towards a party offering radical solutions against other races. E.g. German “Aryans” joining
Nazi party.
7. Gender - Females more likely to lean towards liberal ideology.
8. Education - School syllabus plays an important role. A Chinese may find Indian democracy as
repulsive, because he has been taught that Mao’s Communist ideology is best.

Media’s role in shaping political attitudes:


1. Many people rely on the media as a source of information without even thinking whether it is
true or not. Citizens learn about politics and government primarily from television and
newspapers;
2. Media outlets can influence voters not only through the slant of a particular report, but also
merely by choosing which stories to cover
3. The media has a huge impact in shaping the public opinion of the masses. They can form or
modify the public opinion in different ways depending of what is the objective.
4. The candidates that can pay for more Tv and media exposure have more influence on public
opinion and thus can receive more votes.
Ex: Social media has played a big role in America’s presidential election.

Created with by ForumIAS.com – The Knowledge Network for Civil Services. Visit http//forumias.com New!
http://forumiasacademy – Prelims & Mains Test Series

ForumIAS
MAINS MARATHON COMPILATION August, 2018

Q.10) Discuss the reasons that Prohibition of Employment of Manual Scavengers and Their Rehabilitation
Act 2013 came into being about 25 years ago but manual scavenging continues to be rampant in India.
Suggest some measures for the same.(GS-2)

Answer: Recognising the rights of individuals working as manual scavengers to a rightful, respectful
and dignified life, the Act prohibits the employment of manual scavengers, the manual cleaning of
sewers and septic tanks without protective equipment, and the construction of insanitary latrines. It also
seeks to rehabilitate manual scavengers and provide for their alternative employment.

Why Manual Scavenging still continues?


1. As it is a caste-based profession in Indian society, people belonging to certain castes continue to do
it for lack of other skills. Also, people from other castes do not take up such jobs due to the
stigma attached to them.
2. Failure to recognise the people – under the act, government recognised 12,742 manual scavengers in
13 states. According to Census of India 2011, there are 740,078 households across the country
where human excreta is removed by a person from a dry latrine(this excludes septic tanks,
sewers, railway platforms). Socio-Economic Caste Census 2011 said that there are 182,505 families
in rural India engaged in manual scavenging.
3. The idea of manual scavengers is usually the image of a man inside a septic tank. Women in the
profession are ignored than their male counterparts.
4. There’s no government will to break the caste system.
5. Existence of primitive “insanitary latrines” without water, where the excreta must be physically
removed.
6. According to IndiaSpend, about 12.6% urban households and as many as 55% rural households
in India defecate in the open.
7. The compensation given for rehabilitation is insufficient to pull an individual out of his
profession for decades.

Measures to eliminate Manual Scavenging:


1. Ministry of Social Justice and Empowerment has recognised a new survey process to count
manual scavengers through self-declaration camps.
2. Increasing the rehabilitations amount and providing alternative skills and employment
opportunities.
3. Speeding up toilet construction under Swachh Bharat Abhiyan and eradicating open defecation.
4. Stricter law enforcement to penalise for violating the provisions of law.
5. National Commission for Safai Karamcharis Jagadish Hiremani has directed municipal
authorities to book contractors who violated the Prohibition of Employment as Manual
Scavengers and their Rehabilitation Act, 2013 and officials who failed to enforce it.

Created with by ForumIAS.com – The Knowledge Network for Civil Services. Visit http//forumias.com New!
http://forumiasacademy – Prelims & Mains Test Series

ForumIAS
MAINS MARATHON COMPILATION August, 2018

Q.11) What were the reasons for which Fundamental Duties have repeatedly been called ineffective?
Highlight the need and importance of incorporating FDs in Indian Constitution. (GS-2)

Answer: 42nd Constitutional Amendment to the constitution added a new part IVA to the Constitution.
This part contained the Fundamental duties.

Why FDs are called ineffective:


1. They are non-justiciable as the constitution does not provide for their direct enforcement by the
courts.
2. There is no legal sanction against their violation life in the case of Fundamental Rights.
3. The list of duties is not exhaustive as it does not cover other important duties like casting vote,
paying taxes, family planning and so on.
4. Some of the duties are vague, ambiguous and difficult to be understood by the common man.
They are used to abuse people who choose to interpret the duties differently.
5. Though Swaran Singh Committee had suggested for penalty or punishment for the non-
performance of Fundamental Duties, it was not incorporated.

Need for incorporating FDs in constitution:


1. Swaran Singh committee felt that citizens should become conscious that in addition to the
enjoyment of rights, they also have certain duties to perform as well.
2. They serve as a reminder to the citizens that while enjoying their rights, they should also be
conscious of duties they owe to their country, their society and to their fellow citizens.
3. They serve as a source of inspiration for the citizens and promote a sense of discipline and
commitment among them. They create a feeling that the citizens are not mere spectators but
active participants in the realisation of national goals.

Importance of FDs in constitution:


1. Cherishing noble ideals of freedom struggle is a moral precept and respecting the Constitution,
National Flag and National Anthem is a civic duty.
2. They contain a codification of tasks integral to the Indian way of life.
3. They serve as a warning against the anti-national and antisocial activities like burning the
national flag, destroying public property and so on.
4. They help the courts in examining and determining the constitutional validity of a law. In 1992
SC ruled that in determining the constitutionality of any law, if a court finds that the law in
question seeks to give effect to a FD, it may consider such law to be ‘reasonable’ in relation to
Article 14 (equality before law) or Article 19 (six freedoms) and thus save such law from
unconstitutionality.
5. The moral value of fundamental duties would be not to smother rights but to establish a
democratic balance by making the people conscious of their duties equally as they are conscious
of their rights.

Created with by ForumIAS.com – The Knowledge Network for Civil Services. Visit http//forumias.com New!
http://forumiasacademy – Prelims & Mains Test Series

ForumIAS
MAINS MARATHON COMPILATION August, 2018

Q. 12) The Punjab Cabinet’s decision to amend the law to make acts of sacrilege against the holy books of
major religions punishable with life imprisonment is retrograde and fraught with undesirable
consequences. Explain. (GS-2)

Answer: The law will introduce a new section 295-AA in the IPC after India’s ‘Blasphemy law’, Section
295-A, which criminalises “deliberate and malicious acts intended to outrage religious feelings”.

How is it retrograde:
1. It uses religious sensitivities to score political points.
2. Section 295-A itself already covers it.
3. Providing for a life term for the same offence in relation to religious texts would be grossly
disproportionate.
4. ‘Sacrilege’ itself is a vague term, and would render the section too broad.
5. There are many instance of misuse of laws aimed to protect religious sentiments.
6. It recognises the inherent rise of intolerance in managing conflicting situations between two
groups.
7. The amendment is trying to criminalise blasphemy in a country which is supposed to be secular
and progressive. Even when the restrictions on free speech were introduced in the Constitution,
blasphemy was not one of them.

What could be the consequences:


1. The freedom of speech could be curtailed and anyone claiming to be outraged can now pursue
vexatious prosecutions.
2. It may increase demands for similar legislation in the rest of India to meet demands of different
groups.
3. In the Shreya Singhal case, SC struck down Section 66A of IT Act for being vague and unclear
and being an unreasonable restriction on the right to freedom of speech and expression (Article
19(1)(a)).
4. Punishing sacrilege runs contradictory to the right to privacy – since even private expressions of
discontent against a particular text could be considered sacrilegious.
5. Blasphemy laws are abused by governments to silence opposition, critics and dissidents.
6. Religious extremists have exploited such laws to justify attacks on minorities.

Q.13) Explain what are the individual responsibilities and collective responsibilities with reference to
Council of Ministers? Also highlight the differences between ‘council of ministers’ and ‘cabinet’? (GS-2)

Answer: Responsibility of Ministers


Collective Responsibility
1. This is the fundamental principle underlying parliamentary system of government. Art 75 clearly
states that CoM is collectively responsible to LS.
2. All the ministers own joint responsibility to LS for all their acts of omission and commission.
3. They work as a team and swim or sink together.

Created with by ForumIAS.com – The Knowledge Network for Civil Services. Visit http//forumias.com New!
http://forumiasacademy – Prelims & Mains Test Series

ForumIAS
MAINS MARATHON COMPILATION August, 2018

4. The principle of collective responsibility also means that Cabinet decisions bind all cabinet
ministers even if they differed in cabinet meeting. It is the duty of every minister to stand by
cabinet decisions and support them both within and outside Parliament.

Individual Responsibility
1. Article 75 also contains principle of Individual Responsibility.
2. It states that ministers hold office during pleasure of president – President can remove a minister
even when CoM enjoys the confidence of LS. President removes a minister only on the advice of
PM.
3. In case of a difference of opinion or dissatisfaction with the performance of a minister PM can ask
him to resign or advice President to dismiss him.
4. By exercising this power PM can ensure the realisation of the rule of CR.

Created with by ForumIAS.com – The Knowledge Network for Civil Services. Visit http//forumias.com New!
http://forumiasacademy – Prelims & Mains Test Series

ForumIAS
MAINS MARATHON COMPILATION August, 2018

Q.14) The Constitution itself declares that the Directive Principles of State Policy (DPSP) are fundamental
to the governance of the country. In the light of the statement, discuss the importance of DPSP and
highlight how these are different from the Fundamental Rights? (GS-2)

Answer: The DPSP are enumerated in Part IV of the Constitution. Article 37 of the constitution says that
these principles are fundamental in the governance of the country and it shall be the duty of the State to
apply these principles in making laws.

Importance of DPSP:
1. Directives have great value because they lay down that the goal of Indian polity is ‘economic
democracy’ as distinguished from ‘political democracy’.
2. They remind governments of the basic principles of the new social and economic order, which
the Constitution aims at building.
3. They have helped the courts in exercising their power of judicial review, that is, the power to
determine the constitutional validity of a law.
4. They facilitate stability and continuity in domestic and foreign policies in political, economic and
social spheres in spite of the changes of the party in power.
5. They are supplementary to the fundamental rights of the citizens. They are intended to fill in the
vacuum in Part III by providing for social and economic rights.
6. Their implementation creates a favourable atmosphere for the full and proper enjoyment of the
fundamental rights by the citizens. Political democracy, without economic democracy, has no
meaning.
7. They enable the opposition to exercise influence and control over the operations of the
government. The Opposition can blame the ruling party on the ground that its activities are
opposed to the Directives.

How DPSP differ from Fundamental Rights:

Created with by ForumIAS.com – The Knowledge Network for Civil Services. Visit http//forumias.com New!
http://forumiasacademy – Prelims & Mains Test Series

ForumIAS
MAINS MARATHON COMPILATION August, 2018

Q.15) RCEP ( Regional Comprehensive Economic Partnership) is not just beneficial for its member
countries, but also for the whole of Asia. Analyze. Also, discuss India’s concerns with RCEP. (GS-2)

Answer: RCEP was built upon existing ASEAN+1 FTAs to strengthen economic linkages and enhance
trade and investment.

Beneficial for members:


1. RCEP will provide a framework aimed at lowering trade barriers and securing improved market
access for goods and services for businesses.
2. Asean businesses would follow one set of procedures instead of having to navigate through five
different sets of rules when trading with its RCEP partners. Thus it strengthens trade relations
with FTA partners.
3. It recognises the ASEAN Centrality in emerging regional economic architecture.
4. Enhances transparency in trade and investment relations between participating countries.
5. Facilitation of SMEs’ engagements in global and regional supply chains by enabling SMEs to cope
with challenges arising from globalisation and trade liberalisation.
6. Greater ease of doing business in the region and increase Asean’s attractiveness as a trade and
investment destination.

Beneficial for Asia:


1. Strengthen global production networks and make Asia the world’s factory.
2. Trade barriers in Asia will come down and the new rules will be consistent with WTO
agreements.
3. In the area of investment rules, where no WTO agreement exists, the RCEP will promote easier
FDI flows and technology transfers by multinational corporations.

India’s concerns:
1. NITI Aayog said that India should rethink joining RCEP as it will be disastrous to provide more
market access to China.
2. India’s inability to negotiate a good services deal.
3. India already has bilateral FTAs with Asean, Korea and Japan and negotiations are underway
with Australia and New Zealand.
4. It imposes TRIPS-plus conditions on IPRs.
5. Strategically, this may convey a shift towards China and raise concerns with our partner, US.

Q.16) Malnutrition has emerged as a major challenge in India’s growth story. Discuss the challenges in
eradicating the problem, enumerating the governmental interventions in this regard. (GS-2)

Answer: Malnutrition involves a dietary deficiency. According to WHO, malnutrition is the gravest
single threat to global public health.

Malnutrition challenge to India’s growth:


1. It leads to wasting and stunting in children. This affects their health and thus their educational
outcomes.
Created with by ForumIAS.com – The Knowledge Network for Civil Services. Visit http//forumias.com New!
http://forumiasacademy – Prelims & Mains Test Series

ForumIAS
MAINS MARATHON COMPILATION August, 2018

2. Large numbers of Indian women are anaemic. This results in deaths at the time of child birth.
Also, malnourished mothers are more likely to have underweight children, who will in turn have
a higher risk of physical and cognitive impairment.
3. It wastes precious human resources who eventually contribute to economic growth of the
country. This is due to low-level skilled workforce.
4. Overweight and obesity increase the risk of type 2 diabetes, hypertension, stroke and other
lifestyle diseases and burdening the economy with health costs.
5. Malnourished adults are less able to work, contribute to local economies, and provide care for
their families.
6. Latest figures:
a) 38% of children under 5 are affected by stunting.
b) About 21% of children under 5 are defined as ’wasted’ or ‘severely wasted.
c) 51% of women of reproductive age suffer from anaemia.
d) More than 22% of adult women are overweight.

Challenges in eliminating it:


1. Poverty and lack of proper food availability. Rising costs of food due to poor management of
agriculture policies.
2. Limitations of schemes life PDS in providing quality nutrition.
3. Lack of awareness on micronutrient deficiencies. Low awareness regarding nutrition and use of
local nutritious food including sources of nutrients.
4. Changing lifestyles leading to bad food choices and obesity. Food patterns shifted towards
cereal-centric diets.
5. Inadequate access to safe drinking water and proper sanitation facilities.
6. Inappropriate and sub-optimal infant and young child feeding and caring practices.

Governmental interventions:
1. Mid Day Meal scheme
2. ICDS – Integrated Child Development Service Programme is a programme under which a
package of integrated services consisting of supplementary nutrition, immunization, health check
up are provided to the most vulnerable groups within children and women
3. POSHAN Abhiyaan targets to reduce stunting, undernutrition, anemia (among young children,
women and adolescent girls) and reduce low birth weight by 2%, 2%, 3% and 2% per annum
respectively.
4. Mothers Absolute Affection (MAA) – It is an attempt to bring undiluted focus on promotion of
breastfeeding.
5. National Iodine Deficiency Disorders Control Program (NIDDCP)
6. Rajiv Gandhi Scheme for Empowerment of Adolescent Girls (RGSEAG) – ‘SABLA’
7. ‘Kishori Shakti Yojana’ (KSY) is a special intervention devised for adolescent girls using the ICDS
infrastructure.
8. Weekly Iron and Folic Acid Supplementation (WIFS) Program
9. National Food Security Mission (NFSM) targeted additional production of 25 million tonnes of
food grains.

Created with by ForumIAS.com – The Knowledge Network for Civil Services. Visit http//forumias.com New!
http://forumiasacademy – Prelims & Mains Test Series

ForumIAS
MAINS MARATHON COMPILATION August, 2018

Q.17) Do you think adultery law in India is in contravention to the Constitutional Provisions under Art 14
and 15. Critically analyze, with special emphasis on various governmental reports and committees’ views
on the issue. (GS-2)

Answer: Adultery refers to extra-marital voluntary sexual intercourse. Provisions in Section 497 of IPC
mandates a punishment of 5 years in prison for the man who had consensual sexual intercourse with a
married woman. But it does not punish the woman who had consensual sexual intercourse with a
married man.

Contravention to Art 14 and 15:


1. As it considers that women could not be punished as they were vulnerable.
2. By punishing only a man for adultery, the law is discriminating against citizens based on “sex”,
covered under Article 15.
3. Section 198 (1) of CrPC says only “aggrieved party” can bring forth a complaint on cases of
adultery. Section 198 (2) clarifies that only the husband can be the “aggrieved party”.
4. It is discriminatory against men as it presumes the morality of man and judges his behavior. It is
in violation of Article 14 and 15 which prohibits discrimination on grounds of religion, caste, sex.

Committees:
1. The 42nd Law Commission of India Report and the Malimath Committee on Criminal Law
Reforms suggested that the section be amended to ensure both the erring wife and the husband
be held equally liable for adultery.
2. Law Commission rendered liable only the male offender considering the condition of women in
this country and law’s duty to protect it.
3. SC in Yusuf Abdul Aziz v. The State of Bombay (1954) case held that Section 497 did not violate
the right to equality in Articles 14 and 15 of Constitution. Constitution itself provides for special
provisions with regard to women and children.
4. Fifth Law Commission made suggestions as to changes in the provision, including making the
law gender-neutral.

Q.18) Discuss the effectiveness of Maintenance & welfare of parents’ senior citizen Act, 2007 in dealing
with problems faced by elderlies in India.(GS-2)

Answer: The Maintenance and Welfare of Parents and Senior Citizens Act, 2007 makes it a legal
obligation for children to provide for their elderly parents. Senior citizens (over 60 years of age) who
cannot maintain themselves from their own earning or out of the property owned by them, are entitled to
make an application under the maintenance law.

How the law is beneficial:


1. Cases of people abandoning their elderly parents have significantly risen in the last few decades.
2. It provides speedy and inexpensive mechanism for the protection of life and property of the older
persons.
3. It provides certain rights to the parents to demand for their welfare.

Created with by ForumIAS.com – The Knowledge Network for Civil Services. Visit http//forumias.com New!
http://forumiasacademy – Prelims & Mains Test Series

ForumIAS
MAINS MARATHON COMPILATION August, 2018

4. The rapid changes in social scenario and the emerging prevalence of nuclear family set-ups in
India in recent years made the elderly people exposed to emotional, physical and financial
insecurity.

Challenges in the implementation of law:


1. The system lacks speedy remedial measures to make sure that no one is deprived of their rights.
2. Parents generally do not raise their voices if they face harassments or neglect from their wards. A
very few cases are being reported.
3. Even the compensation for maintenance of elders is very low.
4. The guardians appointed for taking care may often end up harming the elders.
5. Very small no of individuals are aware of the law and the benefits they can avail under its
provisions.
6. The family members, whether close or distant did not approve of approaching the tribunal for
redress since it spoils the reputation of the family as well as their children.
7. Most of the times, the final decision of the tribunal authority is not implemented as they are not
responsible for the implementation of the decision.
8. As the coordination between the departments of revenue, social welfare and police is poor at the
ground level, the implementation of the act faced challenges.

Q.19) The lateral entry scheme, if implemented properly, may foster more competitive spirit, break the
complacency of the higher civil servants and eventually prove to be a pioneering initiative in public
interest. Elucidate. (GS-2)

Answer: Lateral Entry allows for professionals from diverse sectors to enter the civil services at a
secretary level. This is expected to bring in efficiency through competition in the system

How it can help:


1. By bringing in competition with professionals or specialists, the government servants are
compelled to update their knowledge.
2. Civil services in India is a generalist job whereas rapidly changing socio-economic-technological
scenario demands one to be a specialist in various domains. Lateral entry brings in subject
specialists to the job and contributes in improving the efficiency of the system
3. Several committees on Personnel Administration have recommended lateral entry to improve the
efficiency of Indian civil services.
4. Higher bureaucracy in the secretariat often has to examine proposals received from specialised
departments/corporations life the Central Public Works Department, Central Water Commission
and prepare a cohesive note to facilitate the Minister concerned. This needs expertise in the
respective domains.
5. It supplements the existing stock of talent by attracting fresh blood into the system.
6. It can also address the challenges of politics-bureaucrats nexus that is leading to cases of collusive
corruption in India.

Created with by ForumIAS.com – The Knowledge Network for Civil Services. Visit http//forumias.com New!
http://forumiasacademy – Prelims & Mains Test Series

ForumIAS
MAINS MARATHON COMPILATION August, 2018

7. It checks the automatic mode of every member of the higher services reaching the top echelons
and brings in merit in promotions too.
8. There are challenges, however, like the likely induction of loyalists to the current dispensation.
Doubts have been expressed if private business houses would “plant” their people in order to
influence government policies.

Q.20) There needs to be a road map to rescue private Industrial Training Institutes from their weak
state. Explain. Also suggest measures for skills development in India. (GS-2)

Answer: The recent parliamentary report on private ITIs has exposed a scam on the Quality Council of
India’s approval for thousands of private ITIs. The number of private ITIs has grown from under 2,000 to
over 11,000 in five years. Sharda Prasad committee points to the poor state of skill training going on in
these institutes.

Other Problems with private ITIs:


1. Failure to align with global standards.
2. Lack of regulation and monitoring to keep a check on the quality of training.
3. Policy changes make the governmental affiliation mandatory to start operation and ministry
often introduces new conditions.
4. The complex set of rules, regulations and laws governing the sector makes it difficult to setup
and run an institute in India.
5. Huge entry restrictions life working capital requirements.
6. Poor results in the form of placements of trained candidates. This can be due to lack of industry
tie ups and also due to adoption of poor skill standards.

Measures for skill development in India:


1. More industry – institute tie ups can be promoted to cater to the skills demanded by industry.
2. Simplifying red tape to enable private sector participation in developing skills.
3. Separate stream for vocational education (in secondary education), creating vocational schools
and vocational colleges for upward mobility, and having a Central university to award degrees
and diplomas. China has such a separate stream after nine years of compulsory schooling, and
half the students choose VET at the senior secondary level.
4. The Sharada Prasad Committee had recommended that the number of SSCs should correspond
to the National Industrial (Activity) Classification.
5. Enhance employer ownership, responsibility in skill training.
6. Periodic surveys through National Sample Survey Office, to collect data on skill providers and
skill gaps by sector can guide evidence-based policy-making.

Created with by ForumIAS.com – The Knowledge Network for Civil Services. Visit http//forumias.com New!
http://forumiasacademy – Prelims & Mains Test Series

ForumIAS
MAINS MARATHON COMPILATION August, 2018

Q.21) Highlight the need for Universal Social Security Scheme in India. Discuss the major government
interventions for old ages in India. (GS-2)

Answer: Recently, the govt has proposed a comprehensive social security system to provide retirement,
health, old-age, disability, unemployment and maternity benefits to the 500 million workers.

Need for a universal social security scheme in India:


1. India’s workforce stands at 450mn of which around 10% belong to the organised sector and get
social security of some sort. Every year, more than 10 million people are added to the workforce
but most don’t get even the minimum wage and lack any kind of social security coverage.
2. About 83% of the employed persons are in unorganised sector. The labour law coverage for
unorganised sector is lacking in the country.
3. A number of existing schemes already exist on insurance, disability benefits etc run by both the
central as well as state governments.
4. Agricultural labourers are not covered under any kind of social security so far.
5. Lack of a social security measure pushes people into poverty while facing any kind of emergency.

Government interventions for old aged:


1. Rashtriya Vayoshri Yojana (RVY) – All unclaimed amounts from small savings accounts, PPF &
EPF are to be transferred to this fund. Aids and assistive living devices are provided to senior
citizens belonging to BPL category who suffer from age related disabilities such as low vision,
hearing impairment, loss of teeth and locomotor disabilities.
2. Indira Gandhi National Old Age Pension Scheme (IGNOAPS) – It extends social assistance for
poor households- for the aged, widows, disabled and in the case of death of the breadwinner.
Under this scheme, financial assistance is provided to person of 60 years and above and
belonging to family living below poverty line.
3. The Pradhan Mantri Vaya Vandana Yojana(PNVVY) – On payment of an initial lump sum
amount, subscribers will get an assured pension based on a guaranteed rate of return of 8% per
annum payable monthly/quarterly/half-yearly/annually.
4. National Program for the Health Care of Elderly (NPHCE) – to address various health related
problems of elderly people.

Q.22) Street Vendors Act (Protection of Livelihood and Regulation of Street Vending), 2014 aimed at
changing the status quo of street vendor but the implementation gap are defeating the very purpose of
the Act. Discuss. (GS-2)

Answer: Street Vendors Act was brought with the intention of regulating the profession and
guaranteeing the street vendors certain amount of social security.

Provisions:
1. The Bill aims to protect the livelihood rights of street vendors as well as regulate street vending
through demarcation of vending zones, conditions for and restrictions on street vending.
2. Any person intending to undertake street vending needs to register with the Town Vending
Committee (TVC). He may then apply for a vending certificate that will be issued based on
various criteria.

Created with by ForumIAS.com – The Knowledge Network for Civil Services. Visit http//forumias.com New!
http://forumiasacademy – Prelims & Mains Test Series

ForumIAS
MAINS MARATHON COMPILATION August, 2018

3. The state government shall frame a scheme for street vendors. The local authority shall, in
consultation with the planning authority, frame a street vending plan once every five years.
4. The TVC comprises of the municipal commissioner, representatives of street vendors, local
authority, planning authority, local police, resident welfare association and other traders
associations.

Challenges in implementation:
1. Many states did not form vendor committees at the district and city levels. In the absence of rules,
town vending committees are formed unscientifically.
2. Many states are yet to notify the rules. The delay has put on hold any welfare initiatives for street
vendors.
3. Vendors are often at the mercy of municipal corporations which resort to road expansions.
Though the Act prohibits authorities from evicting street vendors until and unless the survey and
ID card distribution are completed, vendors’ associations have accused the authorities of evicting
them.
4. While the Street Vending Act asks planning laws to take cognizance of the requirements of street
vending and align state planning laws to vending needs, little has been done in practice to
achieve this.
5. It also needs to be examined whether some vending spaces can be made available for multiple
use, allowing more than one vendor in different time slots. Eight-hour slots can be another
plausible solution to enable a larger number of vendors to earn their living.
6. Improved work conditions for street vending must be put in place. Vendors should have access
to facilities such as safe drinking water, hygienic toilets, electricity and storage facilities. These
amenities will not only increase the productivity of the vendors but also help in maintaining
sanitary conditions in the area.

Q.23) Discuss the factors behind poor health of the private Industrial Training Institutes of India? In your
opinion what measures should be taken to improve their quality? (GS-2)

Answer: The recent parliamentary report on private ITIs has exposed a scam on the Quality Council of
India’s approval for thousands of private ITIs. The number of private ITIs has grown from under 2,000 to
over 11,000 in five years. Sharda Prasad committee points to the poor state of skill training going on in
these institutes.

Other Problems with private ITIs:


1. Failure to align with global standards.
2. Lack of regulation and monitoring to keep a check on the quality of training.
3. Policy changes make the governmental affiliation mandatory to start operation and ministry
often introduces new conditions.
4. The complex set of rules, regulations and laws governing the sector makes it difficult to setup
and run an institute in India.
5. Huge entry restrictions life working capital requirements.

Created with by ForumIAS.com – The Knowledge Network for Civil Services. Visit http//forumias.com New!
http://forumiasacademy – Prelims & Mains Test Series

ForumIAS
MAINS MARATHON COMPILATION August, 2018

6. Poor results in the form of placements of trained candidates. This can be due to lack of industry
tie ups and also due to adoption of poor skill standards. Placement in NSDC training has been
less than 15%.
7. Lack of a regulator with teeth, for skill development has led to poor quality affiliation,
assessment and certification.
8. A national survey by the research institute (NILERD) of Planning Commission in 2011 about
private ITIs: they offered training in less than five trades; had fewer classrooms and workshops
for practice; and their teachers were very poorly paid.
9. NILERD nationwide survey, 2011 found that ITIs have many internal issues such as staffing and
salaries.

Measures for improving the quality of ITIs:


1. More industry – institute tie ups can be promoted to cater to the skills demanded by industry.
2. Simplifying red tape to enable private sector participation in the skill development industry.
3. The Sharada Prasad Committee recommended that the number of SSCs should correspond to the
National Industrial (Activity) Classification.
4. Periodic surveys through National Sample Survey Office, to collect data on skill providers and
skill gaps by sector to guide evidence-based policy-making.
5. We need better oversight, with a national board for all skill development programmes. The core
work of accreditation, assessment, certification and course standards cannot be outsourced.
6. Mandatory rating system for the ITIs can be published periodically. A ranking of ITIs on several
parameters such as the National Assessment and Accreditation Council in tertiary education can
be replicated.
7. We need a unified national vocational system where the ITIs, NSDC private vocational trainers
work with each other. 12th Five Year Plan recommends a national vocational act that replaces all
scattered regulations.
8. Need to reskill ITI teachers and maintain the student-teacher ratio.
9. Since technology obsolescence is a continuous challenge, financial support envisaged through
NSDC should be extended to the ITIs.
10. A reimbursable industry contribution (RIC) — a 1-2% payroll tax that will be reimbursed when
employers train using public/private infrastructure and provide data. It is implemented in 62
other countries and was recommended in the 12th Plan

Q.24) World has changed and multilateral institutions now have to embed these changes. In the light of
the statement, highlight the importance of WTO in the present world order and needs to reform it. (GS- 2)

Answer: There is an increasing threat to multilateral institutions from newly emerging forces of global
politics and economy.

Where multilateral institutions need to change?


Reforms in multilateral institutions life UN, WB, IMF and WTO are demanded. These demands arise
from the following reasons:
1. The socio-political-economic reasons behind their formation no longer prevail in the world.
Created with by ForumIAS.com – The Knowledge Network for Civil Services. Visit http//forumias.com New!
http://forumiasacademy – Prelims & Mains Test Series

ForumIAS
MAINS MARATHON COMPILATION August, 2018

2. After the fall of USSR, there is a tendency towards unilateralism. With growing developing
economies, world is becoming increasingly multilateral.
3. Lack of adequate representation to countries of the third world. Most of the times powers are
concentrated in Europe and North America.

Importance of WTO
1. It helped developing countries like India and China gain from global trade.
2. It gives voice to the small and developing countries in trade disputes with the developed nations.
3. It puts forward a trade agenda that is fair, equitable and rules-based. Thus it reduces scope for
tariffs and non tariff barriers that obstruct trade.
4. WTO’s Most Favoured Nation and national-treatment articles stipulate that each WTO member
must grant equal market access to all other members and that both domestic and foreign
suppliers must be treated equally.
5. Rules are designed to help governments resist lobbying efforts by domestic interest groups
seeking special favours.
6. Brings greater certainty and predictability to international markets. It would enhance economic
welfare and reduce political tensions.

Need to reform WTO


1. U.S. has systematically blocked the appointment of new Appellate Body members and de facto
impeded the work of WTO appeal mechanism.
2. Concerns over the politicisation of the Body appointment and reappointment process; and the
quasi-attribution of permanent Appellate Body seats to the U.S. and European Union (EU).
3. The dispute settlement process favours the powerful nation in reality. As the small and
developing countries cannot impose sanctions after failure of all opportunities at dispute
settlement, they end up taking the loss.
4. The Doha Development Agenda is stalled by the attempts of developed countries to push their
own agenda. This is against the interest of developing nations.
5. WTO has played a very limited role in helping address other global issues related to trade, such
as food security, climate change and global trade imbalances.
6. In the light of growing protectionism and emerging Regional Trading groups, WTO should
reinvent itself and be a true representative of all the trading nations.

Q. 25) Highlight the issue faced by children at shelter homes for homeless children or those in conflict
with law. Discuss the provisions of Juvenile Justice (Care and Protection of Children) Act, 2015 that
governs these shelters and suggest some measures to deal with the issues faced by children. (GS-2)

Answer: Juvenile Justice (Care and Protection of Children) Act, 2015 is passed by Parliament of India to
consolidate and amend the law relating to children alleged and found to be in conflict with law and
children in need of care and protection.

Created with by ForumIAS.com – The Knowledge Network for Civil Services. Visit http//forumias.com New!
http://forumiasacademy – Prelims & Mains Test Series

ForumIAS
MAINS MARATHON COMPILATION August, 2018

Issues faced by children at shelter homes:


1. NCPCR submitted that out of a total of 2,874 children’s homes surveyed, only 54 institutions
could be given positive reviews.
2. Staff at India’s orphanages have often been implicated in criminal charges.
3. Children are sexually abused, beaten, and psychologically abused.
4. These homes are not supervised by any regulatory bodies and escape scrutiny.
5. They do not have any standardisation in quality of care.
6. Poor infrastructure - Poor lighting, cramped accommodation, lack of well-trained, educated staff
that treat homeless children reflect the poor state of infrastructure at orphanages.
7. Orphanages are instead ‘ruled’ with fear. Children are also kept unaware of their rights, and who
they can report child rights violations to.

Provisions of the law:


1. The Child Care Institutions in respect of children in conflict with law are the Observation Home,
Special Home, Place of Safety and fit facility.
2. Mandatory registration of Child Care Institutions - All child care institutions, whether run by
State Government or by voluntary or non-governmental organisations are to be mandatorily
registered under the Act.
3. Registration applications of Child Care Institutions are to be disposed of within six months
otherwise it would be considered as dereliction of duty and will invite departmental
proceedings.

Measures to deal with the issues:


1. Strengthen regulation and monitoring of the quality of these shelter homes.
2. Periodic audits by NCPCR and due debate along with follow-up on the recommendations of the
commission.
3. Mechanisms should be put in place for children to complain directly to the concerned officials.
4. Mandatory registration of the homes and stringent checks on their necessary infrastructure.
5. Advanced child care methods and psychological development should be prioritised to just
providing them the basic needs of shelter and food.

Q.26) Discuss the controversial issues around article 35A of the Indian constitution. Should Supreme
Court repeal this provision? Provide valid arguments in support of your opinion. (GS-2)

Answer: 35A : Article 35 A empowers the Jammu & Kashmir State Legislature to define, “permanent
residents” and to protect their rights in terms of employment, property and scholarship or aid by
government organization.

What are the controversies around it:


1. Article 35 A was added to the Constitution of India through a Presidential Order. Article 35A is
based on a pact between the Union and the State and cannot be unilaterally altered.
2. It is alleged that the law promotes gender bias and discrimination. It restricts the right to
property if a native woman marries a man not holding a permanent resident certificate.

Created with by ForumIAS.com – The Knowledge Network for Civil Services. Visit http//forumias.com New!
http://forumiasacademy – Prelims & Mains Test Series

ForumIAS
MAINS MARATHON COMPILATION August, 2018

3. It is argued that the article goes against the “very spirit of oneness of India” as it creates a “class
within a class of Indian citizens”.

Why it needs to be repealed?


1. The parliamentary route of lawmaking was bypassed when the President incorporated Article
35A into the Constitution. Though SC in Puranlal Lakhanpal judgement says that President may
modify an existing provision in the Constitution under Article 370, it is silent whether President
can introduce a new Article.
2. Restricting citizens from other States from getting employment or buying property within Jammu
and Kashmir is a violation of fundamental rights under Articles 14, 19 and 21 of the Constitution.
Why it needs to be retained?
1. By striking down Article 35A, it would allow people from outside Jammu-Kashmir to settle in the
state and acquire land and property, and the right to vote, thus altering the demography of the
Muslim-majority state.
2. The matter is “very sensitive” and requires a “larger debate”.
As the matter pertains to autonomy of the state of J&K, it needs to be debated more and more
consultations with people of the state can be made. SC cannot impose any decision on the people and it is
better if any decision comes from the legislature.

Q.27) India’s law governing motor vehicles and transport is archaic, lacking the provisions necessary
to manage fast motorisation. Critically analyze the provisions of Motor Vehicles (Amendment) Bill.
(GS-2)

Answer: The Motor Vehicle Act of 1988 has not been updated ever since it first came into being. Latest
Road Accidents in India report by Road Transport and Highways Ministry reports 4,80,652 accidents in
India in 2016 leading to about 1.5 lakh deaths. The new Motor Vehicle Amendment Bill aims to curb
accidents with stiffer penalties.

MV Act: Where does it lack?


1. Passenger transport sector within cities has grown amorphously, with vested interests exploiting
the lack of transparency and regulatory bottlenecks.
2. State-run services have not kept pace with the times. Major investments made in the urban metro
rail systems are yielding poor results in the absence of last-mile connectivity services.

Provisions of MV(Amdt) Bill


1. Setting up National Road Safety Board.
2. Defines digital aggregators life Uber, Ola and Meru as market place for a passenger to connect
with driver.
3. Opens up the public transport sector for the private players.
4. Aims to create National Transportation Policy within the ambit of a national Act giving power to
the Centre.

Created with by ForumIAS.com – The Knowledge Network for Civil Services. Visit http//forumias.com New!
http://forumiasacademy – Prelims & Mains Test Series

ForumIAS
MAINS MARATHON COMPILATION August, 2018

Pros & Cons


Benefits from the bill:
1. Creating an equitable regulatory framework for the orderly growth of services is critical.
2. Regulatory changes introduced in Europe over the past few years for bus services have fostered
competition, reduced fares and increased services operating across European Union member-
states.
3. The effort to curb institutionalised corruption at Regional Transport Offices by making it possible
for dealers to directly register new vehicles, and enabling online applications for driving licences
is welcome.
4. New provisions to harness technology, including CCTV monitoring, to improve road safety are
appreciated.

Opposed on these grounds:


1. That it would threaten federalism because central law will prevail over all states and ‘transport’ is
a subject in concurrent list.
2. Amendments dealing with road safety are likely to achieve little without strong enforcement by
the States.
3. That dealers may end up gaining much from these new amendments.
4. Lack of a professional accident investigation agency to determine best practices.
5. Public Transportation System at present is a state subject and is operated by State Transport
Undertakings (STUs). The transport sector workers fear if the private sector is allowed to
interfere in this sector, it will create more difficulty to the public and will be a threat to the job
security of the employees working under STUs.
6. Feared that it will expedite privatization of public transport system and take-over by big private
corporates leading to monopolization and enhancement of the public transport fares to the
detriment interest of the common working people.

Q.28) Tinkering with the RTE Act without sufficient thought will erode a major constitutional
achievement. Discuss.(GS-2)

Answer: RTE Act: The Right of Children to Free and Compulsory Education (RTE) Act, 2009 is the
consequential legislation envisaged under Article 21-A. Under this, every child has a right to full time
elementary education of satisfactory and equitable quality in a formal school which satisfies certain
essential norms and standards.

The proposed amendments aims to give States the power to detain students who fail an examination in
Class 5 or 8.

How would it impact the outcomes:


1. It would weaken one of the progressive features of the RTE Act, which is to guarantee the
continued presence of the child in school during the formative learning phase.

Created with by ForumIAS.com – The Knowledge Network for Civil Services. Visit http//forumias.com New!
http://forumiasacademy – Prelims & Mains Test Series

ForumIAS
MAINS MARATHON COMPILATION August, 2018

2. It will allow State Boards to declare a student failed and detain her on the basis of an
examination, although Section 30(1) of the RTE Act holds out the assurance that no child shall be
required to face any Board examination till completion of elementary education.
3. The concerns on learning outcomes produced by India’s schooling system are determined not
only by a student’s effort but also by the number and quality of teachers, processes for
continuous assessment and engagement of parents and the community in encouraging
excellence.
4. In 2016 NITI Aayog found that bringing back detention in elementary schooling would increase
the dropout rate, impacting the poor and Dalits the most as they depended on government
institutions.
5. When parents are unable to ensure regular attendance of children due to social circumstances,
detaining them for non-performance may not act as an incentive to attend school regularly.

Q.29) What is meant by social constraints? Are constraints of any kind necessary for enjoying
freedom? What is the difference between the negative and positive conception of liberty? (GS-2)

Answer:

Social constraints : Given the diverse interests and ambitions of people any form of social living requires
some rules and regulation. These rules may impose some constraints to on the freedom of individuals
and are called social constraints.

Constraints are needed for freedom:


1. Some constraints may free us from insecurity and provide us with the conditions in which we can
develop ourselves.
2. We need some constraints or else society would descend into chaos. Differences may exist
between people regarding their ideas and opinions, they may have conflicting ambitions, they
may compete for scarce resources.
3. Sme legal and political restraints ensure that differences may be discussed and debated without
one group coercively imposing its views on the other

Constraints not needed for freedom:


1. In South Africa, the constraints imposed by the apartheid regime discriminated between citizens
based on their race.
2. Constraints on freedom from social inequality like in the caste system and based on economic
inequalities are not desirable.
3. Constraints limit an individual from enjoying equality of status and opportunity.
4. Also state most of the times limits the freedom of individual in the name of larger public interest.
Here the check should be the extent of compromise of individual freedom and the importance of
national interest. The debate between Fundamental Rights and DPSPs is an example.

Created with by ForumIAS.com – The Knowledge Network for Civil Services. Visit http//forumias.com New!
http://forumiasacademy – Prelims & Mains Test Series

ForumIAS
MAINS MARATHON COMPILATION August, 2018

As freedom is at the core of human society, is so crucial for a dignified human life, it should only be
constrained in special circumstances. The ‘harm caused’ must be ‘serious’.

Difference between +ve and -ve liberty:

Negative liberty’ seeks to defend an area in which the individual can ‘do, be or become’ whatever he
wants. This is an area in which no external authority can interfere. It recognises that human nature and
dignity need an area where the person can act unobstructed by others.

‘Positive liberty’ is concerned with looking at the conditions and nature of the relationship between the
individual and society. It recognises that one can be free only in society (not outside it) and hence tries to
make that society such that it enables the development of the individual.

Q.30) Union Cabinet has approved an amendment to The Scheduled Castes and the Scheduled Tribes
(Prevention of Atrocities) Act to undo ‘dilution’ of the law by the Supreme Court. In the light of the
statement discuss the changes made by Supreme Court to SC/ST act along with the reasons given by it
and the recent changes proposed by government to it. (GS-2)

Answer: What is SC judgement:


1. Public servants could be arrested only with the written permission of their appointing authority,
while for private employees, Senior Superintendent of Police concerned should allow it.
2. A preliminary inquiry should be conducted before the FIR was registered to check if the case fell
within the ambit of the Act, and whether it was frivolous or motivated, the court ruled.

What is the amendment to SC/ST (Amendment) Act?


The bill will make three changes to SC/ST (Prevention of Atrocities) Act, 1989:
1. Set aside SC order that an FIR should be lodged only after a preliminary inquiry is conducted by
a gazetted officer. It will insert section 18A that states that preliminary enquiry is not needed for
FIR and immediate arrest of a person.
2. Say that no approval of a senior official would be needed for arrests
3. Say that no provision of section 438 of criminal procedure court would be applicable. Section 438
of CrPC states that only after preliminary enquiries find sufficient ground for lodging of an FIR,
this would be done. This was made applicable after SC judgement. Govt wants it to revert back.

Why did the SC recommend those changes?


1. To protect people against arbitrary arrests under the Act.
2. There were “instances of abuse” of the Act by “vested interests” for political or personal reasons.
3. One could not be arrested without following established procedure, and doing so would violate
right of life and liberty.

Why did the govt want to change?


1. Alleged potential of misuse would not deserve to be considered as a valid, justifiable or
permissible ground for reading down stringent provisions of the PoA (Prevention of Atrocities)
Act, 1989.
Created with by ForumIAS.com – The Knowledge Network for Civil Services. Visit http//forumias.com New!
http://forumiasacademy – Prelims & Mains Test Series

ForumIAS
MAINS MARATHON COMPILATION August, 2018

2. There were 47,338 cases of crime against members of the SC/ST in 2016. In the face of growing
atrocities against SCs/STs, aggrieved persons may now think twice before registering a
complaint, as the process is backed by checks which prevent the automatic arrest of an accused.
3. The changes focus on protecting the liberty of the accused and can dilute the rights of the
complainant.
4. While the SC/ST (Prevention of Atrocities) Act provided a legal cover to numerous”atrocities”
that SCs and STs suffer, in reality, its implementation on the ground remained weak.

Q.31) Health care in India is obviously not egalitarian and becoming increasingly dependent on the
private sector to deliver health care. Discuss and suggest measures for equitable medical care. (GS-2)

Answer: Of the total health spending in India, the government contributes just 29%. In the UK, the
government’s share is 83%.

The health care services in India are not egalitarian. The out-of-pocket expenses for medical care are
about 70% of all medical expenditure, enough to push a family into poverty.

Problems with present healthcare system:


1. Poor government spending in healthcare is resulting in deteriorated standards in service
provision from public sector. Indian government only spends about 1.4% of the GDP on the
sector
2. The gap between what is technologically possible and what government hospitals generally
provide widened appreciably after the technological leaps in medical care – this can be called
“medical rationing”.
3. The deteriorating state of public healthcare in India means that an increasingly larger percentage
of its citizens are being driven to the private sector for their medical needs.
4. Mounting costs and decreasing availability of drugs is a major issue
5. Medical rationing has other detrimental effects like creating distrust of the public in government
hospitals.

Measures for equitable health care:


1. The new Ayushman Bharat health scheme to provide secondary and tertiary care to those who
are socioeconomically deprived.
2. NITI Aayog’s document recommends the government to prioritise preventive care rather than
provide curative care.
3. AIIMS like institutions in every state to make tertiary care accessible.
4. Generic medicines should be mandated and be made available.
5. Niti Aayog suggested a model that provides for a greater role for private players in the India’s
healthcare sector.

Created with by ForumIAS.com – The Knowledge Network for Civil Services. Visit http//forumias.com New!
http://forumiasacademy – Prelims & Mains Test Series

ForumIAS
MAINS MARATHON COMPILATION August, 2018

Q.32) ‘BRICS has grown in influence in its first decade but is still far from achieving its initial goals’.
Discuss (GS-2)

Answer:

BRICS is the acronym for an association of five major emerging national economies: Brazil, Russia, India,
China and South Africa. It represents about 40% of the world’s population and 22% of global GDP.

How it grew in influence:


1. New Development Bank is established to provide an alternative to IMF and WB led global
multilateral financing.
2. BRICS summit has been successful in terms of building a coalition of emerging markets that want
to save multilateral trade regime
3. BRICS Business Council has been actively enhancing trade and economic cooperation in diverse
sectors
4. “BRICS Plus” which began at Xiamen summit is a great opportunity for interaction among
leaders.

Challenges in achieving its goals:


1. Reform of global financial governance is still pending with NDB playing a limited role only.
BRICS recently reiterated that IMF needs to better represent developing nations
2. Democratisation of the United Nations is resisted by the powerful nations trying to retain their
control.
3. Expansion of the Security Council — because two of its members China and Russia do not want
the other three members (India, South Africa and Brazil) to obtain parity in the global pecking
order.
4. Rising protectionism is affecting rules-based multilateral trading order.
5. China’s dominance is growing and China-Russia proximity is a reality.
6. BRICS Partnership on New Industrial Revolution (PartNIR) can make a contribution only if it
goes beyond the ministries of industry and engage with private sector.
7. Though BRICS outreach to Africa began in 2013, it has picked up momentum now. African
nations need big loans from the New Development Bank (NDB) for their infrastructure projects.
8. China introduced the “BRICS Plus” format at the Xiamen summit last year. The precise role of
“BRICS Plus” is yet to be understood.
9. No decision was taken to set up the BRICS credit rating agency that India favours.

Q. 33) Blasphemy law is not required in a secular state like India. Critically examine, with special
emphasis on SC judgements in this regard. (GS-2)

Ans 1. Blasphemy refers to a matter pertaining to lack of respect toward God, religion, a religious icon, or
something else considered sacred.Section 295A of IPC is a cognizable offence which penalizes any

Created with by ForumIAS.com – The Knowledge Network for Civil Services. Visit http//forumias.com New!
http://forumiasacademy – Prelims & Mains Test Series

ForumIAS
MAINS MARATHON COMPILATION August, 2018

deliberate insult to a religion, its beliefs or any class of citizens. Recently,Punjab government proposed to
add Section 295AA in IPC to expand and tighten law on Sacrilege.

Why is Blasphemy law not required in India?


1. India already has constitutional provisions under Articles 25-28 which protect religion and its
sanctity.
2. Art 19(1)(a) has reasonable restrictions to protect any speech which may hurt religious
sentiments.
3. Blasphemy has been described as irreverence towards God or Religion, however the term
“Religion‟ itself lacks a proper definition for itself.
4. It is considered to be against the spirit of Fundamental right of Speech and Expression. USA has
no blasphemy law.
5. Blasphemy law is incompatible with the UN Covenant on Civil and Political Rights
6. Blasphemy law is often used as a tool by the majority to oppress the minority.
7. The allegations of blasphemy have been used by vigilante groups and non-state actors to justify
and instigate incidents of interreligious violence
8. Blasphemy law is required by the states that have an official religion like Pakistan

However, there are reasons which favor introduction of blasphemy law in India:
1. To inculcate value of respect for sanctity of the God, Religion and Religious beliefs.
2. Religion affects the actions of human beings. Legal protection provided by state to protect
religious belief and sentiments leads to stable society and governance.
3. Most of the countries implement Blasphemy laws as a reasonable restriction for the maintenance
of communal harmony.
4. Section 295A was also introduced to control series of communal violence Various SC
judgements regarding blasphemy:

Ramji Lal Modi v. State of Uttar Pradesh


• The case challenges the constitutional validity of law.
• Case argue that Art. 19(2) puts only reasonable restrictions but section 295A casts its net much
wider by criminalizing all speech that was intended to outrage religious feeling
• A five-judge bench of the Supreme Court upheld the constitutionality of Section 295 A
• Supreme court observed that section 295A did not cover all types of Insults but only intentional
insult

The Superintendent, Central Prison, Fatehgarh v Ram Manohar Lohia


• Under this case Supreme Court observed that speech which is prohibited should have a direct
connection with disruption of public order. It should not just be a remote connection
• This is contrary to previous judgment (Ram ji Lal Modi case) which gave the order that a slight
connection of freedom of speech with public disorder falls under Section 295 A

Created with by ForumIAS.com – The Knowledge Network for Civil Services. Visit http//forumias.com New!
http://forumiasacademy – Prelims & Mains Test Series

ForumIAS
MAINS MARATHON COMPILATION August, 2018

Conclusion

On the basis of above arguments, it is clear that India has a strong constitutional and legal framework to
protect the sanctity of religion, its idols, beliefs and sentiments. However, it is pertinent to note that in the
interest of maintenance of public order, it is necessary to arm the civil authorities with required legal and
rule based powers. Thus, a blasphemy law catering to the needs of the society may be the order of the
day. However, care must be taken to ensure that it has built-in provisions to protect the minorities from
exploitation.

Q.34) Discuss the recent controversy around collegium system of Supreme Court. What is the criterion
to decide seniority in the Supreme Court. Is there there any need to bring changes to this criterion?
(GS-2)

Answer: After the Three Judges Cases, a judge is appointed to the supreme court by the president on the
recommendation of the collegium which is a closed group of Chief Justice of India, the four most senior
judges of the court and the senior-most judge hailing from the high court of a prospective appointee.

Controversy – the lowering in seniority of one of the three new judges in the government notification on
the appointments despite collegium recommending the name ahead.

Criteria to decide seniority:


1. Decided on the basis of date of induction in the Supreme Court. A judge who takes oath earlier
becomes senior to another who takes oath later.
2. Where warrants for appointment of judges to Supreme Court are issued by the government on
different dates, the seniority is automatically decided by virtue of the dates of swearing-in by the
CJI.
3. There is no stated rule in the Memorandum of Procedure (MoP) to decide the seniority of judges
whose warrants of appointment are issued on the same date. As the warrants are issued by the
government in a sequence, the practice has been for the CJI to administer the oath in the same
order. For example, the warrants for appointment of current CJI Misra and now retired Justice J
Chelameswar were issued on the same day but, as Misra’s warrant was numbered above that of
Justice Chelameswar, he was sworn in first. This ensured that he became CJI, deemed as senior to
Justice Chelameswar.
4. These are on the basis of the recommendation of the Collegium, which comprises the five most
senior SC judges. The Collegium’s recommendations for any name can be returned by the
government, but if the Collegium reiterates the name, the government is bound to issue the
warrant of appointment. The procedure for this is laid down in the MoP.

Need to bring changes:


1. It leads to government deliberately slighting the judiciary.
2. This seniority affects the chances of a judge becoming the CJI as in the case of Justice
Chelameswar.
3. The seniority of a judge also affects the membership of the Collegium.

Created with by ForumIAS.com – The Knowledge Network for Civil Services. Visit http//forumias.com New!
http://forumiasacademy – Prelims & Mains Test Series

ForumIAS
MAINS MARATHON COMPILATION August, 2018

Q.35) India and Pakistan will resume their talks on various aspects of the Indus Waters Treaty. Discuss
the provisions of Indus water treaty and highlight the point of contentions between India and Pakistan
over it. (GS-2)

Indus Waters Treaty is a water-distribution treaty between India and Pakistan, brokered by the World
Bank to use the water available in the Indus System of Rivers

Provisions:

According to this agreement, control over the water flowing in three “eastern” rivers of India — the Beas,
the Ravi and the Sutlej was given to India, while control over the water flowing in three “western” rivers
of India — the Indus, the Chenab and the Jhelum was given to Pakistan.
Since Pakistan’s rivers receive more water flow from India, the treaty allowed India to use western rivers
water for limited irrigation use and unrestricted use for power generation, domestic, industrial and non
consumptive uses such as navigation, floating of property, fish culture, etc. while laying down precise
regulations for India to build projects
Since the ratification of the treaty in 1960, India and Pakistan have not engaged in any water wars. Most
disagreements and disputes have been settled via legal procedures, provided for within the framework of
the treaty. The treaty is considered to be one of the most successful water sharing endeavours in the
world today
Both countries agreed to exchange data and co-operate in matters related to the treaty. For this purpose,
treaty creates the Permanent Indus Commission, with a commissioner appointed by each country. It
would follow the set procedure for adjudicating any future differences and disputes arising over the
allocation of waters. The Commission has survived three wars and provides an ongoing mechanism for
consultation and conflict resolution through inspection, exchange of data and visits. The Commission is
required to meet at least once in a year to discuss potential disputes as well as cooperative arrangements
for the development of the Indus System of Rivers.
Either party must notify the other of plans to construct any engineering works which would affect the
other party and to provide data about such works.
The annual inspections and exchange of data continue, unperturbed by tensions on the subcontinent.
In cases of disagreement, Permanent Court of Arbitration (PCA) or a neutral technical expert is called in
for arbitration.

point of contentions
The treaty has not considered Gujarat state in India as part of the Indus river basin. The Indus river is
entering the Rann of Kutch area and feeding in to Kori Creek during floods.
Without taking consent from India, Pakistan has constructed Left Bank Outfall Drain (LBOD) project
passing through the Great Rann of Kutch area with the assistance from the world bank.
Water released by the LBOD is enhancing the flooding in India and contaminating the quality of water
bodies which are source of water to salt farms spread over vast area
Indus originates from Tibet in China. If China decides to stop or change the flow of the river, it will affect
both India and Pakistan.

Created with by ForumIAS.com – The Knowledge Network for Civil Services. Visit http//forumias.com New!
http://forumiasacademy – Prelims & Mains Test Series

ForumIAS
MAINS MARATHON COMPILATION August, 2018

Climate change is causing melting of ice in Tibetan plateau, which scientists believe will affect the river in
future.
India completed the building of the Kishanganga dam in Kashmir and continued work on the Ratle
hydroelectric power station on the Chenab River despite Pakistan’s objections

Q.36) Provide an account of the evolution of Draft National Register of Citizens and discuss the
controversies and challenges surrounding it? (GS-2)

Answer: National Register of Citizens: The National Register of Citizens (NRC) is the list of Indian
citizens of Assam. The stated aim of the updating exercise is to identify “illegal immigrants” in the state.

Evolution of NRC:
1. It was prepared in 1951, following the census of 1951.
2. The first pilot project for updating the National Register of Citizens was launched in 2010 after
protests against illegal migrants in electoral rolls in 1997 elections.
3. The process of NRC update was taken up in Assam as per a Supreme Court order in 2013 in
order to wean out cases of illegal migration from Bangladesh and other adjoining areas. It was
carried out under The Citizenship Act, 1955, and according to rules framed in the Assam Accord.

Controversies surrounding it:


1. Bengali Muslims are the community most often branded as illegal Bangladeshi immigrants. They
felt that they were put under greater scrutiny than other groups.
2. The sudden appearance of an “original inhabitants” category and they being subjected to less
rigorous scrutiny is another issue.
3. The whole process is accused of arousing communal tensions along Hindu-Muslim lines.
4. Also this may strain relationships with Bangladesh as the people identified as illegal immigrants
need to be deported.
5. This may not in any way help in bringing peace to this region already troubled by conflicts
around ethnicities.
6. Neither the state nor the Centre has clarified what happens to those who lose their cases in the
Foreigners’ Tribunals, whether they will be detained, deported or allowed to stay on without the
rights and privileges of citizenship.
7. Foreigners’ Tribunals – they have challenges in ensuring justice due to the brief window of time
given by the border police to produce proof of citizenship to the lack of legal aid.

Detention centers – Those deemed to be foreigners are transferred to detention centres. They have
languished here for years in a legal limbo. Though declared as foreigners foreigners, there is no
repatriation treaty under which they can be deported to Bangladesh.

Created with by ForumIAS.com – The Knowledge Network for Civil Services. Visit http//forumias.com New!
http://forumiasacademy – Prelims & Mains Test Series

ForumIAS
MAINS MARATHON COMPILATION August, 2018

General Studies – 3

Q.1) Discuss the factors behind India’s ailing indigenous defense manufacturing. How new Strategic
Partnership guidelines would be able to transform defense manufacturing in India? (GS-3)

Answer: India continues to remain in the strategically-vulnerable position of being the world’s largest
arms importer, accounting for 12% of the global imports from 2013-2017.

Factors behind poor indigenous industry


1. Ordinance Factories : Dependence only on the government owned defence public sector units
(PSUs) and ordnance factories which failed to produce cutting edge technologies.
2. R&D: Researching and developing new military systems is the job of the Defence Research and
Development Organization (DRDO). The DRDO consumes 20% of the Indian government’s total
R&D funding. Its inherent problems and contradictions led to the gross failure of indigenous
production.
3. Rama Rao Committee on DRDO says that it requires passionate and inspiring leadership. IT
points to poor HR management with only 3% of its scientists have PhDs in engineering-related
subjects. It warns that DRDO’s “biggest challenge” is to attract, nurture and retain talent.
4. Bias against private sector participation – Kelkar committee had suggested nominating more than
a dozen Indian private sector companies Raksha Udyog Ratnas, with a status equivalent to that
of the defence.

Strategic Partnership guidelines – Recently the Defence Acquisition Council (DAC) approved the
implementation of Strategic Partnership guidelines.
1. 4 segments – The model has four segments — submarines, single engine fighter aircraft,
helicopters and armoured carriers/main battle tanks. These would be specifically opened up for
the private sector.
2. Global Equipment Manufacturers – One Indian private company would be selected in each
segment which would tie-up with shortlisted global equipment manufacturers to manufacture
the platforms in India under technology transfer.
3. Procurement – All procurements would be executed by specially constituted Empowered Project
Committees for timely execution.

How they can help:


1. The model aims to revitalise defence industrial ecosystem and build indigenous capabilities in
the private sector to design, develop and manufacture complex weapon systems.
2. They lay emphasis on incentivisation of transfer of niche technology and higher indigenous
content.
3. This creates scope for innovation transfer from foreign companies.
4. Private sector participation can attract more investments into the sector.

Created with by ForumIAS.com – The Knowledge Network for Civil Services. Visit http//forumias.com New!
http://forumiasacademy – Prelims & Mains Test Series

ForumIAS
MAINS MARATHON COMPILATION August, 2018

Q. 2) Power loom sector, which contributes to 40 per cent of the nation’s man-made fabric demand, is
in a shutdown mode in Surat, discuss the challenges faced by the sector and steps taken by
government to deal with them. GS-3
Answer:
The decentralized powerloom sector is the lifeline of Indian Textile Industry. 60% of Indian cloth is
produced through decentralized power loom sector, provides employment to more than 7 million
workers.

Major issues faced by Powerloom sector:


1. Poor infrastructure - Most of the powerloom clusters face common problems like poor roads,
effluent issues, frequent power cuts, non-availability of water, no warehouses & logistic issues
etc.
2. Obsolete Technology - Modernisation in looms is less, There are less than 15,000 modern looms,
whereas traditional looms are in large numbers.
a. Value addition and the manufacturing of fabrics according to customer’s compliances, is
not possible due to obsolete technology of looms.
b. Loom’s depreciation & maintenance cost is very high. It results in increase in the
operating cost and the overall cost of production also increases.
c. The productivity of plain powerloom is very low as compared to automatic,
shuttleless and semi-automatic looms
Unskilled labour - Most of the labors lack good technical knowledge. They are not been trained for
increasing productivity & efficiency.
Lack of marketing support - Mostly, the weavers are uneducated or less educated and are unable to
estimate the demand of their products according to the need of the market.
d. The middlemen exploit the powerloom owners and gain the main surplus.
e. The merchants and agents are united while the powerloom owners are dispersed. So, the buyers have
more bargaining power and sellers are dependent on the buyers.
Finance - The majority of the weavers start their business with the self-finance. They don’t
have adequate supply of finance to modernize their power looms. They don’t have sufficient capital
to pile up the stock of raw material when the prices are low.

Government initiatives:
1. Powerloom Development and Export Promotion Council (PDEXCIL) - nodal agency for
development and promotion of exports of powerloom products by promotional activities such as
participation in international trade shows, sending and hosting trade delegations, and sustained
image-building exercises through advertisements abroad, publications and audio visuals.
2. Comprehensive Powerloom Cluster Development Scheme (CPCDS) by Ministry of Textiles - for
development of Powerloom Mega Clusters.
3. Telangana State government announced incentives, provided training and marketing facilities
and had been promoting handloom textiles in a big way by placing bulk orders for school
uniforms, Ramzan gifts and Bathukamma sarees.

Created with by ForumIAS.com – The Knowledge Network for Civil Services. Visit http//forumias.com New!
http://forumiasacademy – Prelims & Mains Test Series

ForumIAS
MAINS MARATHON COMPILATION August, 2018

Q.3) Enumerate the land-use categories as maintained in the Land Revenue Records and highlight the
patterns of land use changes that have taken place in last few decades in India. (GS-3)

Answer:

Land use categories mentioned :

The land-use categories as maintained in the Land Revenue Records are as follows :
1. Forests : Area under actual forest cover is different from area classified as forest. There may be an
increase in this category without any increase in the actual forest cover.
2. Barren and Wastelands : The land which may be classified as a wasteland such as barren hilly
terrains, desert lands, ravines, etc. normally cannot be brought under cultivation with the
available technology.
3. Land put to Non-agricultural Uses : Land under urban and rural settlements, infrastructure,
industries etc., are included in this category.
4. Area under Permanent Pastures and Grazing Lands : Most of this type land is owned by the
village ‘Panchayat’ or the Government. Only a small proportion of this land is privately owned.
5. Area under Miscellaneous Tree Crops and Groves : The land under orchards and fruit trees are
included in this category.
6. Culturable Wasteland :Any land which is not cultivated for more than five years is included in
this category.
7. Current Fallow : it is the land which is left without cultivation for one or less than one
agricultural year.
8. Fallow other than Current Fallow : This is also a cultivable land which is left uncultivated for
more than a year but less than five years.
9. Net Area Sown: The physical extent of land on which crops are sown and harvested is known as
net sown area

Broad patterns of land use changes:


1. The rate of increase is the highest in case of area under non-agricultural uses. This is due to the
changing structure of Indian economy, which is increasingly depending on the contribution from
industrial and services sectors
2. The increase in the share under forest could be by increase in the demarcated area under forest
rather than an actual increase in the forest cover
3. The increase in net area sown is a recent phenomenon due to use of culturable waste land for
agricultural purpose.
4. As the pressure on land increased from agricultural and nonagricultural sectors, the wastelands
and culturable wastelands have witnessed decline over time.
5. The decline in land under pastures and grazing lands can be explained by pressure from
agricultural land.

Created with by ForumIAS.com – The Knowledge Network for Civil Services. Visit http//forumias.com New!
http://forumiasacademy – Prelims & Mains Test Series

ForumIAS
MAINS MARATHON COMPILATION August, 2018

Q.4) Critically analyse the government’s decision to turn down the foreign assistance for disaster relief
with special reference in case of Kerala. (GS-3)

Answer: Why govt turned down the assistance:


1. To demonstrate India’s superpower status. India should demonstrate that it had the strength to
withstand and counter calamities and also help its neighbours, as in the case of December 2004
tsunami and piracy attacks in the Indian Ocean.
2. This would strengthen its case for seeking to be a permanent member of United Nations Security
Council and also hasten the prospect of superpower status by 2020.
3. The fear of foreign hand, the spies who would come with it and interfere in the country’s internal
affairs.

Merits:
1. India has been the biggest contributor to the UN Development Programme and the biggest
recipient of assistance. It is a party to the regulations of the UN and its conditionalities for
assistance.

Demerits:
1. This time should be used to use all assistance, Indian and foreign, to rebuild Kerala.
2. This is an unnecessary distraction for the Central and State governments at a time of a grave
crisis.
3. The quoted amounts anyway will not meet even a fraction of the cost of rebuilding the
infrastructure and bringing the State to normalcy.
4. Bilateral and multilateral assistance will take a long time in coming though accepted. So any
decision in this regard should be quickly taken.
5. The policy of not accepting foreign assistance has not taken India even one step towards fulfilling
its superpower ambitions.
6. India was given admission into Australia Group and Wassenaar Arrangement it did not want,
and Missile Technology Control Regime because its system of missile control was unmatchable,
but the membership of NSG and the APEC were denied.
7. The fear of spies is no more relevant. The development of technology is such that foreigners do
not need to come in hordes to India to know what is happening or to influence decision-making.
8. This may also have a negative impact on India’s relations with the UAE.

Q.5) What is GST? What are the challenges in its implementation? What are the possible solutions? GS 3
• GST is an indirect tax reform which aims to remove the tax barriers between states and create a
single market.
• It is a single tax on the supply of goods and services, right from the manufacturer to the
consumer.
• The Government had introduced the 122nd Amendment Bill, 2014, in the Parliament to facilitate
the introduction of GST in the country.
• The Bill was finally passed by both the Houses in 2016
• It is a consumption based tax/levy. It is based on the “Destination principle.”

Created with by ForumIAS.com – The Knowledge Network for Civil Services. Visit http//forumias.com New!
http://forumiasacademy – Prelims & Mains Test Series

ForumIAS
MAINS MARATHON COMPILATION August, 2018

• GST is applied on goods and services at the place where final/actual consumption happens.
• It came into force from 1 July, 2017.
• It is levied at multiple rates ranging from 0% to 28%.

There are three components of GST:-


Central GST (CGST) – it will be Levied by Centre
State GST (SGST) – It will be levied by State
Integrated GST (IGST) – It will be levied and collected by Central Government on supply of goods and
services

What are the various challenges in implementation of GST?


• Financial challenges: GST is expected to cause a downfall in state revenue and the bill ensuring
compensation from the union government has still not been passed.
• Federal system: The states would lose their autonomy to levy indirect taxes and will be totally
dependent on the centre government.
• Administrative challenges: some states are demanding control over taxing all businesses. It is
contentious issue as the central government also needs funds for its policies and for
compensating the states.
• GST Council: With one-third voting share in the hands of Union government, may states feel the
share of the states should be more.
• No parliamentary approval is needed for GST rates. The Central GST Bill, 2017 allows the central
government to notify CGST rates, subject to a cap. This implies that the government may change
rates subject to a cap of 20%, without requiring the approval of Parliament.
• Lack of skilled manpower to effectively migrate from older system to GST
• The requirement of e-way bills for inter-State movements has also been a cause of concern.

Effects on States
• According to the Reserve Bank of India (RBI), even as the fiscal position at the Centre remains
stable (Central budget deficit for 2017-18 pegged at 3.2% of gross domestic product), there has
been a marked deterioration in the gross fiscal deficit of states.
• The figure for 2016-17 is not finalized yet but could be as high as a deficit of 3.4%.
• Revenue expenditure of the states has risen sharply in recent years with greater financial
devolution and increased expenditure.
• In aggregate, the states spend about 30% more than the Centre. This gap will further increase
with GST.
• The GST is a destination-based tax, and as such is viewed as being to the advantage of the
consuming States and to the detriment of the producing States.
• However the formula for compensating to states for such loss has been devised in GST.

What are the suggestions?


• It may be worth reconsidering these rates and bringing them down to the 5 per cent slab for
stronger linkages between farmers and the food processing industry and creating jobs in rural
areas.

Created with by ForumIAS.com – The Knowledge Network for Civil Services. Visit http//forumias.com New!
http://forumiasacademy – Prelims & Mains Test Series

ForumIAS
MAINS MARATHON COMPILATION August, 2018

• Since the raw material could be sourced directly from farmers instead of being entirely
depending on middlemen in mandis, e-NAM provides this opportunity to graduate to a real pan-
India market for agricultural products.
• GST would ensure that farmers in India, who contribute the most to GDP, will be able to sell their
produce for the best available price.
• A smooth GST regime can break inter-state barriers on movement and facilitate direct linkages
between processors and farmers. This can transform the operations of mandis too if other
necessary reforms to free up agricultural markets are undertaken.

Q.6) State the advantages and disadvantages of using Coal bed methane as a fuel in India. What is the
present scenario of CBM resources in India? GS-3

Advantages of CBM as a fuel


• CBM is an environmentally safe gas: Methane has been labelled as a Green House Gas (GHG) by
United Nations Framework Convention on Climate Change (UNFCCC).
• Using CBM as a fuel will halt methane emission into environment and thus reducing emission of
greenhouse gas from coal mining.
• Extraction of CBM prior to coal mining activities makes mining activities safer by degassing the
coal seams.
• Extraction of CBM would help in increasing the domestic gas production. Currently, contribution
of CBM to domestic natural gas production is 1.6%.

Disadvantages related to Coalbed Methane


• Despite the huge reserves, a mismatch exists between estimated resources and gas in-place.
• The following are some of the issues with respect to the simultaneous operations of Coal Bed
Methane and coal mining by multiple owners:
• There is a possibility of damage of gas wells resulting in explosive atmosphere in coal mines
during simultaneous extraction of coal and CBM in the same vertical boundary by two different
owners.
• Multiple ownership for simultaneous exploitation may not be desirable for the life, health and
safety of the workers employed in such mines.
• Simultaneous operation over the same leasehold area requires the development of Safe Operating
Procedures (SOP) for each operation based on assessments of risks

Present scenario of CBM resources in India


• The government has identified 26,000 square km of area for CBM operation with total estimated
CBM Resources of 2,600 billion cubic meters (91.8 TCF).
• India has fourth largest proven coal reserves in the world.
• It has significant prospects for exploiting CBM.
• The Ministry of Petroleum and Natural Gas in consultation with Ministry of Coal has identified
around 26,000 sq.km area for the operation of CBM.

Created with by ForumIAS.com – The Knowledge Network for Civil Services. Visit http//forumias.com New!
http://forumiasacademy – Prelims & Mains Test Series

ForumIAS
MAINS MARATHON COMPILATION August, 2018

Q.7) Examine the issue of man-animal conflict in India. Also, evaluate the government’s strategy to
resolve the problem. (GS-3)

Answer: Data shows that about one person has been killed every day for the past three years by roaming
tigers or rampaging elephants.

Reasons for the issue:


1. Road Kills – traffic density is growing and some of this road network is being expanded through
protected areas.
2. Livestock grazing – in forests leads to human-wildlife conflict as carnivores are attracted towards
the easy prey and become direct enemies of livestock graziers.
3. The increasing human population led to expansion in urban and rural parts and in peripheries of
protected areas.
4. Humans encroach on clear large areas of protected forest patches for development of agricultural
and horticultural lands.
5. Fragmentation of large habitats and shrinking sizes of forest patches makes landscape unviable
for wild animals as their needs are not fulfilled.

Government’s strategy:
1. Adequate habitat to wild animals
2. Maintenance of physical barriers to wild animals
3. Active guarding of crops
4. Stopping of degradation of habitat quality
5. Providing adequate and immediate compensation to affected families.
6. India has national parks, sanctuaries, biosphere reserves, protected forests and conservation areas
where hunting of animals is prohibited. Most of these protected areas in India are too small for
territorial animals such as the tiger to roam about.
7. Though there has been an increase in wildlife sanctuaries in India, their overall area has
considerably reduced. Maharashtra got five additional sanctuaries between 2006 and 2014. But
there is a 50 % reduction in the total area.
Most of the solutions show a knee jerk approach in handling a complex developmental problem. The
solutions can include:
1. improvement of habitat to augment food and water availability and to minimize the animal
movement from the forests to the habitations.
2. essential to train the police offices and local people.
3. sensitize the people about the Do‘s and Don‘ts to minimize conflicts
4. Providing technical and financial support for development of necessary infrastructure and
support facilities
5. eco-development activities in villages around Protected Areas to elicit cooperation of local
community in management of the Protected Areas.

Created with by ForumIAS.com – The Knowledge Network for Civil Services. Visit http//forumias.com New!
http://forumiasacademy – Prelims & Mains Test Series

ForumIAS
MAINS MARATHON COMPILATION August, 2018

Q.8) Wind energy is India’s biggest source of renewable electricity by far, what are the factors
contributing to this achievement? Discuss the hurdles that wind energy sector is facing in faster
development of energy generation? (GS-3)

Answer: Wind power accounts for nearly 10% of India’s total installed power generation
capacity. India’s total wind energy capacity now stands at 34,042 MW. The wind power potential of the
country has been assessed to be 302 GW

Factors for success of wind energy in India:


1. Geographical terrain of India – with mountains, valleys and coasts favors windmills
2. Offshore wind power generation ability due to long coastline
3. Cost competitiveness, mature wind energy value chain and technology-edge
4. Government is playing an active role in promoting the adoption of renewable energy resources
by offering various incentives, such as generation-based incentives (GBIs), capital and interest
subsidies.

Hurdles faced by wind energy sector:


1. Grid connectivity remains a challenge as wind power is generated in hilly and remote areas.
2. Falling tariffs and renegotiation of agreements with state governments
3. Solar energy is given more priority these days
4. Challenges of environmental damage in biodiversity rich areas life Western Ghats and Himalayas

Q.9) Why does equality matters? How do we distinguish between political, economic and social
dimensions of equality? (GS-3)

Answer: The preamble of Indian constitution guarantees its citizens equality of status and opportunity.

Importance of equality:
1. Equality is very important because our society did not practice equal access in the past. The
practice of untouchability is one of the crudest manifestations of inequality.
2. Equality makes India a true democracy by ensuring equality of dignity and status.
3. Equal opportunity means that government can implement special schemes and measures for
improving the conditions of certain sections of society: children, women, and socially and
educationally backward classes.
4. It invokes the idea that all human beings have an equal worth regardless of their colour, gender,
race, or nationality.
5. Some differences which are considered natural need no longer be seen as unalterable. For
instance, advances in medical science and technologies have helped many disabled people to
function effectively in society. Today, computers can help blind

Dimensions of equality:
• Political Equality – Political equality would normally include granting equal citizenship to all
the members of the state. Equal citizenship brings certain basic rights such as the right to vote,
freedom of expression, movement and association and freedom of belief. These are rights which
Created with by ForumIAS.com – The Knowledge Network for Civil Services. Visit http//forumias.com New!
http://forumiasacademy – Prelims & Mains Test Series

ForumIAS
MAINS MARATHON COMPILATION August, 2018

are considered necessary to enable citizens to develop themselves and participate in the affairs of
the state.
1. Social Equality – Political equality only legally guarantees access to goods. But pursuit of
equality requires that people belonging to different groups and communities also have a fair and
equal chance to compete for those goods. For example, some of the customs deny women equal
rights of inheritance.
Economic Equality – Economic inequality exists in a society if there are significant differences in wealth,
property or income between individuals or classes. Equal opportunities available to people would those
who have talent and determination the chance to improve their condition.

Q.10) States need to reconsider their stand against Kasturirangan committee report on Western ghat in
the wake of recent calamity. Comment (GS-3)

Answer: The monsoon floods in Kerala and parts of Karnataka would have been averted had the
recommendations of the expert committee on Western Ghats been implemented.

Kasturirangan committee report –


Instead of declaring entire Western Ghats as Ecologically Sensitive Area, the panel said that 90% of the
“natural landscape” should be protected.
1. Economic options should not forbade but answer protection of Western Ghats.
2. Supervising forests and bettering their productivity to ascertain inclusive growth and economical
gains for local communities.
3. Removing the cash crop plantations such as rubber, agricultural fields and settlements should be
from the protection regime.
4. Integrating forest accounts into state and national economic assessments
5. Initiating an ecosystem service fund to help villages around the forests
6. Promoting sustainable agriculture
7. Encouraging ecotourism for local benefits.
8. Establish a Decision Support and Monitoring Centre for Geospatial Analysis and Policy Support
in the Western Ghats to supervise changes and recommend state government on policy reform.
9. High-resolution maps delimiting ecologically sensitive areas, down to each village settlement,
must be put in the public domain so that people can be involved in taking decisions about
environment.
10. A ban on all polluting industries categorised as most hazardous in the Water (Prevention and
Control of Pollution) Act, 1974, and Air (Prevention and Control of Pollution) Act, 1981.
11. The Forest Rights Act, 2006 that recognises the rights of dwellers on forest resources, will be
implemented in letter and spirit and the consent of Gram Sabhas concerned will be mandatory
for any project.
12. Strict regime for Hydro-power projects including cumulative impact assessment of such projects
and ensuring minimum water flow in the rivers in the lean season.

What were the objections:


1. Need to balance human development pressures along with protection of Western Ghats.
Created with by ForumIAS.com – The Knowledge Network for Civil Services. Visit http//forumias.com New!
http://forumiasacademy – Prelims & Mains Test Series

ForumIAS
MAINS MARATHON COMPILATION August, 2018

2. Allowing extractive industries such as quarrying and mining to operate in the region for
industrial and revenue purposes.
3. Calculation of what constitutes the sensitive core and what activities can be carried out.
4. They expressed the view that declaring entire Western ghats region as an ecologically sensitive
zone would seriously impair economic activity.

Why states should reconsider:


1. Sustainable development of the region depends on the health of the ecosystem in this
biodiversity hotspot.
2. Activities affecting the environment need to be reconsidered in the wake of disasters life recent
Kerala floods.
3. Man-made factors worsened the impact of recent unprecedented rain in Kerala and Karnataka.
4. Satellite images confirm the extent of degradation in these ecologically sensitive areas.
5. Sustainable development is an imperative in times of global warming and extreme weather
occurrences.

Q.11) Discuss the roles that telemedicine can play in improving the health conditions of rural India?
What are the challenges in its implementing? (GS-3)

Answer: Telemedicine is the use of telecommunication and information technology to provide clinical
health care from a distance.

Potential role of telemedicine


1. Improved access – The physician population ratio in India is 0.6/1000. This ratio is among the
lowest in the world. Telemedicine reduces the physical barriers in access to doctors.
2. Cost effective – The actual public health spending in India is only 1.04% of GDP. This is grossly
inadequate for the maintenance of good public health. Telemedicine makes medicine more
affordable with the use of technology.
3. Less infrastructure – 68% of the population still lives in the rural areas. Rural health care system
is plagued with several problems from shortage of healthcare professionals and health
infrastructure. About 60-80% of the physician positions in various specialties are vacant in the
rural health care services. Telemedicine reduces the infrastructure needed for delivering the
service.
4. Bridges rural – urban divide in health outcomes: The rural/urban healthcare disparity is reflected
in the healthcare outcomes as IMR in urban population is 27 whereas in rural population it is 44.
Similarly, total fertility rate (TFR) is 1.8 in the urban population whereas in rural population it is
2.6.
5. Telemedicine applications can be classified into two basic types.

a) Store-and-forward or asynchronous type – there is exchange of pre-recorded data between two or


more individuals at different times/places.
b) Real-time or synchronous type – there is a real time exchange of information by methods such as in
videoconferencing.
Created with by ForumIAS.com – The Knowledge Network for Civil Services. Visit http//forumias.com New!
http://forumiasacademy – Prelims & Mains Test Series

ForumIAS
MAINS MARATHON COMPILATION August, 2018

c) These two basic approaches to telemedicine can be applied to a wide array of clinical services,
including disease diagnosis, triage, management and follow up of diseases

Challenges:
1. Lack of basic infrastructure life electricity and communication network to utilise the technology.
2. Lack of digital literacy.
3. Insufficient financing to these projects. The cost of this infrastructure is a big concern as there are
not enough funds.
4. Despite the technology, there is need for other professionals to guide the rural people in
explaining the processes.
5. Linguistic diversity in India is a bigger challenge to improve the accessibility of the technology.

Q.12) Discuss the concept of ‘hothouse earth’. What measures are required to prevent a ‘hothouse
earth’ pathway? (GS-3)

Answer: A “greenhouse Earth” or “hothouse Earth” is a period in which there are no continental glaciers
on the planet. The levels of carbon dioxide and other greenhouse gases are high and sea surface
temperatures (SSTs) range from 28 °C in the tropics to 0 °C in the polar regions.

Measures to prevent hothouse earth pathway:


1. Fundamental shifts in social values and economic mores are essential, like reduced energy
consumption.
2. Increasing contributions from renewables and improvements in energy efficiencies.
3. Technological innovation like green technologies and climate smart agriculture techniques.
4. Increased funding for innovation in climate friendly technologies.
5. Global deals like the Paris Climate Agreement and International Solar Alliance should work
towards greener goals.
6. Creating or enhancing biological carbon stores life afforestation, preserving water bodies.
Biodiversity conservation, improved forest management, and the development of carbon-capture
systems that store carbon underground.

Q.13) TRAI has recently recommended the auction of spectrum for offering 5G services. How is the 5g
technology different to 4g technology and what would be its possible effects on global industries?
(GS-3)

Answer:

5g – The basic performance criteria for 5G systems have been set by ITU in their IMT-2020
recommendation. It provides for speeds up to 20 gigabits per second and has only been demonstrated
with millimeter waves of 15 gigahertz and higher frequency. The more recent 3GPP standard includes
any network using the NR (New Radio) software.

Created with by ForumIAS.com – The Knowledge Network for Civil Services. Visit http//forumias.com New!
http://forumiasacademy – Prelims & Mains Test Series

ForumIAS
MAINS MARATHON COMPILATION August, 2018

Difference between 5g and 4g:


1. Frequencies : One fundamental difference is 5G’s use of unique radio frequencies to achieve
what 4G networks cannot. The radio spectrum is broken up into bands, each with unique features
as you move up into higher frequencies. 4G networks use frequencies below 6 GHz, but 5G will
likely use extremely high frequencies in the 30 GHz to 300 GHz range.
2. Wavelength: 5G also uses shorter wavelengths, which means that antennas can be much smaller
than existing antennas while still providing precise directional control. Since one base station can
utilize even more directional antennas, it means that 5G will support over 1,000 more devices per
meter than what’s supported by 4G.
3. 5G networks will more easily understand the type of data being requested, and will be able to
switch into a lower power mode when not in use or when supplying low rates to specific devices,
but then switch to a higher powered mode for things like HD video streaming.

Feature 4G 5G

Speeds of up to 100 gigabits per second.


Speed 100mbps to 1gbps speed
It is almost 1,000 times faster than 4G

Bandwidth 200mbps >1 gbps

Unified IP and seamless combination of LAN/


technology 4G + WWWW
WAN

Dynamic information access, variable devices.


Dynamic information access, variable devices
SErvice
mainly used for voice, multimedia and internet with all capabilities
over IP based traffic.

Possible impact of 5g on global industry:

1. In the age of Internet of Things (IoT) where a world in which not just people but all things are
connected, 5G is the technological answer. It makes possible billions of new connections, and
making those connections secure and instantaneous.
2. The 5G Global Value Chain, including network operators, core technology and component
suppliers, device OEMs, infrastructure providers, and content and application developers, is
expected to grow output to $3.5 trillion in 2035
3. It offers network operators the potential to offer new services to new categories of users.

Created with by ForumIAS.com – The Knowledge Network for Civil Services. Visit http//forumias.com New!
http://forumiasacademy – Prelims & Mains Test Series

ForumIAS
MAINS MARATHON COMPILATION August, 2018

Q.14) A clampdown on the sale of oxytocin will have severe public health consequences. Critically
analyze. (GS-3)

Answer: Oxytocin:
Oxytocin is a hormone that acts on organs in the body and as a chemical messenger in the brain,
controlling key aspects of the reproductive system, including childbirth and lactation, and aspects of
human behaviour.
Recently the Union Health Ministry imposed a ban on the retail sale and private manufacture of oxytocin

Why it was banned:


1. Because of the misuse of the hormone in the dairy industry. Because oxytocin stimulates lactation
in cattle, dairy farmers inject the drug indiscriminately to increase milk production. This has
spawned several unlicensed facilities that manufacture the drug for veterinary use.
2. Oxytocin leads to infertility in dairy animals.
3. It has also been linked to mastitis, a painful inflammation of the udder. Milk consumers worry
about exposure to it through dairy products
4. There are also concerns that oxytocin can enter the food chain through milk that can be harmful
for human consumption, though it was not established.

Impact of the ban on health:


1. The drug is a synthetic version of a human hormone. It is a lifesaver for women. Doctors use it to
induce labour in pregnant women and to stem postpartum bleeding. WHO recommends it as the

Created with by ForumIAS.com – The Knowledge Network for Civil Services. Visit http//forumias.com New!
http://forumiasacademy – Prelims & Mains Test Series

ForumIAS
MAINS MARATHON COMPILATION August, 2018

drug of choice in postpartum haemorrhage. 45,000 of Indian women die due to causes related to
childbirth each year.
2. There are a few alternatives to oxytocin like misoprostol, but these are even more difficult to get.

Q.15) What are cryptocurrencies? Do you think that cryptocurrencies should be banned? (GS-3)

Introduction
• Recently the government is considering the introduction of a regulatory regime for virtual or
crypto currencies, such as Bitcoin, that would enable the levy of the Goods and Services Tax on
their sale. In the light of this the discussion whether to ban cryptocurrencies has come to light.
Crypto currencies:-
• It is a digital currency which allows transacting parties to remain anonymous while confirming
that the transaction is a valid one. It is not owned or controlled by any institution – governments
or private.
• There are multiple such currencies bitcoin, ethereum, ripple are some of the popular ones.
• Currently, they are neither illegal nor legal in India.

Yes,they need to be banned :-


• Government is wary that regulation will provide legitimacy to “what is currently ambiguous,”
and may lead to further rise in its valuation and end up contributing “to the investment bubble.
• concerns about companies that run MLM and Ponzi schemes in the name of Bitcoin.
• Effective policy to curtail ransomware which has caused havoc in the UK’s National Health
Service and elsewhere would be to ban the use of bitcoin and similar “cryptocurrencies”.
• Anonymity and fungibility have always made cash the favoured tool for illegal activities.
• Credit cards, cheques and bank transfers are too easily traceable. Bitcoin moves the criminal’s
favoured financial tool online, with disastrous consequences.

No:-

• Regulation is fine
• It can be traded on registered exchanges in a bid to “promote” a formal tax base, while
keeping a tab on their use for illegal activities such as money laundering, terror funding
and drug trafficking.
• The lack of regulation or acknowledgment by Indian authorities has led to the Indian
cryptocurrency industry establishing its own self regulatory body and watchdog titled
the Digital Asset and Blockchain Foundation of India (DABFI).
Crypto-currency can also be used for a lot of legal activities such as booking tickets, buying coffee or fast
food, depending of which retailers accept such currency
Banning will give a clear message that all related activities are illegal and will disincetivise those
interested in taking speculative risks.
Banning will impede tax collection on gains made in such activities.
Any decision that ruled the cryptocurrency to be illegal in India would mean that India’s nascent but
growing bitcoin industry will have to shut down.

Created with by ForumIAS.com – The Knowledge Network for Civil Services. Visit http//forumias.com New!
http://forumiasacademy – Prelims & Mains Test Series

ForumIAS
MAINS MARATHON COMPILATION August, 2018

International examples:
• Japan declared Bitcoins as a legal tender in April this year, causing the price of Bitcoins to
spike.
• Australia recently declared that it will accept Bitcoins as legal currency from July 2017
and it will also be exempt from goods and service tax.
• Russia is planning to adopt Bitcoin as a legal payment method in 2018.
Decentralised nature of Bitcoin makes it impractical to ban, and this will encourage the use of
illegal channels like hawala networks.

What can be done?


• Regulating the currency instead would signal a boost to blockchain technology, encourage the
development of a supervision ecosystem (that tracks legal activities and may also assist in
tracking illegal activities) and promote a formal tax base.

Q.16) Discuss the impact of GST in infrastructure sector. Suggest measures to be taken by the government
to boost growth in infrastructure sector in India. (GS-3)

Introduction:
• Infrastructure is the backbone of Indian economy with total infrastructure spending is expected
to be about 10% of GDP in the twelfth five year plan according to the government.
Impact of GST in infrastructure sector :-
Positives:
Earlier dichotomy would vanish:
• The cascading effect of Central and state indirect taxes was a concern.With GST this issue would
fade away.
• Valuation of goods and services in works contracts would now be put to rest as works contracts
would be regarded as supply of services.
• GST is expected to bring in predictability for infrastructure projects.
• Expected to boost the infrastructure sector with the elimination of ´tax on tax’ and the
introduction of input tax credit .

Works contracts and EPC


• The major gain from this treatment is that the tax would be now charged on the actual
contractual base.
• Also, local versus inter-state works contracts, that at present leads to innumerable disputes,
should get eliminated.
• GST is expected to impact cement positively. The overall indirect tax incidence is currently
estimated to be around 25 per cent.
• The GST is expected to enable a reduction in logistics cost by as much as 20 per cent to 30 per
cent, as firms reconfigure their supply chains on four counts.
• The roadways are going to get better with GST. The interstate movement of goods will be
facilitated properly. It will save the unproductive time wasted in the movement of goods.
Created with by ForumIAS.com – The Knowledge Network for Civil Services. Visit http//forumias.com New!
http://forumiasacademy – Prelims & Mains Test Series

ForumIAS
MAINS MARATHON COMPILATION August, 2018

Negatives:

• For project owners, the new legislation may not lead to a conducive future.
• Credit restrictions on works contracts resulting in an immovable property coupled with increase
in GST rates could increase cost outlay.
• Exemptions and concessions to infrastructure have been completely withdrawn. This could also
lead to increased working capital requirements. Project cost could rise due to increased burden of
indirect taxes.
• Electricity being outside the purview of GST, power generation companies would continue to
have indirect taxes as a significant cost factor.
• Flat GST rate of 18% would lead to increased incidence on infrastructure projects.
• The cost of construction services will also be impacted due to credit restrictions provided under
Section 17(5) of the GST Act.

Civil aviation:

Aviation is about to get costly. The credit on tax paid by Airlines will not be credited to them now. The jet
fuel is also going to burn the pockets of Airlines.
The government has taken many measures to boost infrastructure sector like recently by allocating 3.96
lakh crore to the sector.But more is needed.

Measures to be taken by government to boost infrastructure sector:-


• A specific programme for development of multi-modal logistic parks together with multi-modal
facilities need to be drawn up and implemented.
• Government should relook at the Minimum Alternate Tax (MAT) scheme to make infrastructure
sector more lucrative for private players; a lower MAT rate or an alternative should be
considered for mega infrastructure projects.
• Land amendments are crucial for ‘Make in India.

Implement kelkar committee recommendations which suggested


• The government to strengthen three key pillars of a PPP framework
• Also asked for amendments to the prevention of corruption Act setting up of a dedicated
institute for PPP’s, proper risk-sharing measures
• The government needs to give a serious thought and revisit the PPP models and re-instate some
confidence into the private sector to take interest again.
• India needs to develop a better regulatory mechanism, a rational pricing system, reform financial
markets and strengthen dispute resolution mechanisms so that the private sector finds
infrastructure projects economically feasible.

Q.17) Examine the need for increasing public investment to boost growth in India. Also, discuss the
problems associated with public investment in India (GS-3)

Need for public investment :-

Created with by ForumIAS.com – The Knowledge Network for Civil Services. Visit http//forumias.com New!
http://forumiasacademy – Prelims & Mains Test Series

ForumIAS
MAINS MARATHON COMPILATION August, 2018

• According to Keynesian economics that lowering the rate of interest may not do much for private
investment if the expected rate of return is depressed.
• The slowing of both global trade and domestic manufacturing may have had precisely this effect
by lowering the long-term expectations held by private investors. To flag demand India needs
public investment
• Exports have been stagnated for more than a year now and private consumption largely is tied to
income, so public investment remains the only source of demand growth.
• The Indian economy is much more dependent on the global economy which itself is in turmoil so
public investment is the abode.
• Ramping up public investments may be necessary to fill-in for and indeed crowd-in private
investment.
• Public investment will be the main driver of growth as the private sector’s ability to leverage
balance sheets for infrastructure projects is either already over-extended or has very little
headroom.
• Rapidly rising state investment is associated with high economic growth (China and India).
• A policy relying purely on private investment is extremely unlikely to be successful as the
experience of the U.S., Japan and EU confirms.
• International demand has slowed down due to successive crises in the international economy –
the 2008 financial crisis, the sovereign debt crisis in Europe, falling crude oil prices, slowdown in
the Chinese economy and depressed global commodity prices.So relying on this would adversely
affect economy so public investment is needed.
• For setting up industries in backward/tribal areas for better regional development.

Why depending too much on public investment is not good :-


• Strained balance-sheets of companies
• even if demand picks up it is unclear if companies can undertake further investments
RBI reluctant to cut repo rates in spite of low inflation rates.
Problems associated with public investment in India :-
• continuing financial problems of Indian banks and of many large corporations.
• One factor holding back investment is that major reforms, especially regarding land acquisition,
simply have not happened.
• A third factor creating a drag on investment is the real cost of borrowing.
• Increased fiscal deficit and public debt will dent foreign investors confidence.
• Problem of revival of inflation based on public spending.
• The mixed economy character has always been an avowed objective of the government since the
first industrial policy of 1948. The future is likely to see the true emergence of the mixed economy
character of India as both private and public investment would boost the economy.

Created with by ForumIAS.com – The Knowledge Network for Civil Services. Visit http//forumias.com New!
http://forumiasacademy – Prelims & Mains Test Series

ForumIAS
MAINS MARATHON COMPILATION August, 2018

Q.18) While safeguard duty on solar panels from China, Malaysia may benefit domestic manufacturers, it
will do a lot of damage to existing solar projects. Explain. And discuss the other factors that are hurting
the development of solar power in India. (GS-3)

Answer: The government implemented a 25% safeguard duty on solar cell imports from China and
Malaysia for the period between July 30, 2018 and July 29, 2019. This import duty has been placed in
order to encourage local solar panel manufacturers in the country in a push to the ‘Make in India’ effort.
Thus it may encourage local manufacturers of solar panels.

How it damages existing solar projects?


1. Majority of top solar projects are using these imported panels especially from China.
2. More than 10,000 MW capacity of solar panels are imported annually from China and Malaysia
and this duty would adversely impact the commercial viability of some solar power projects
3. The increased tariffs will be ultimately passed on to the customers, hampering the adoption of
clean energy.
4. The duty does not provide any relief to developers in SEZs.
5. It would increase cost of solar power and make it less attractive to buying utilities. Thus it could
jeopardise the pace of growth of development of solar power. Tariffs are likely to rise 30-35 paise
per unit due to the duty.
6. There could also be short term delays in project completion.

Factors hurting development of solar power in India:


1. Solar projects require land and lack of availability of land is a challenge.
2. Lack of grid connectivity to sites of solar power production.
3. Huge dependence on imports of solar panels make the production expensive if there are any
changes in international market.
4. During daytime, the weather may be cloudy or rainy, with little or no sun radiation. Hence, this
makes solar energy panels less reliable as a solution.
5. Solar panels also require inverts and storage batteries to convert direct electricity to alternating
electricity so as to generate electricity. While installing a solar panel is quite cheap, installing
other equipment becomes expensive.
6. Energy production is quite low compared to other forms of energy.
7. The renegotiation of existing Power Purchase Agreements (PPA) in the light of falling solar prices
is a trouble for the producers.

Q.19) DNA Technology (Use and Application) Regulation Bill, 2018 can be an important tool in
solving crimes, but it is important that there are safeguards to protect human rights and prevent
miscarriages of justice. Discuss. (GS-3)

Answer: The primary intended purpose for enactment of “The DNA Technology (Use and Application)
Regulation Bill” is for expanding the application of DNA-based forensic technologies to support and
strengthen the justice delivery system of the country.

Created with by ForumIAS.com – The Knowledge Network for Civil Services. Visit http//forumias.com New!
http://forumiasacademy – Prelims & Mains Test Series

ForumIAS
MAINS MARATHON COMPILATION August, 2018

Role in solving crimes:


1. It is the process of determining an individual’s DNA characteristics, which are as unique as
fingerprints.
2. Although 99.9% of human DNA sequences are the same in every person, enough of the DNA is
different that it is possible to distinguish one individual from another, unless they are
monozygotic (“identical”) twins.
3. DNA profiling uses repetitive sequences that are highly variable, called variable number tandem
repeats (VNTRs). VNTR loci are so variable that unrelated individuals are extremely unlikely to
have the same VNTRs.

In this way, forensic DNA profiling helps in solving cases involving offences affecting the human body
such as murder, rape, human trafficking, or grievous hurt and those against property including theft,
burglary, and dacoity.

The Bill’s provisions will enable the cross-matching between persons who have been reported missing on
the one hand and unidentified dead bodies found in various parts of the country on the other, and also
for establishing the identity of victims in mass disasters.

Need to protect human rights


1. Using DNA effectively during criminal investigations requires proper crime scene examination,
trained and reliable policing, a trusted chain of custody of samples, reliable analysis, and proper
use of expert evidence in court. Without these prerequisites, a DNA database will exacerbate
rather than solve problems in the criminal justice system: for example, by leading to miscarriages
of justice through (false matches or misinterpretation or planting of evidence, and diverting
resources) from more important priorities.
2. Many errors occur before samples get to the laboratory. So the requirement for laboratory
accreditation should include quality assurance for crime scene examination.
3. The Bill’s proposed DNA Regulatory Board is too powerful and insufficiently transparent or
accountable.
4. There is a need to restrict DNA profiling so that it uses only non-coding DNA, a commonly used
international standard. It which prevents the use of parts of the DNA which code for personal
characteristics, including medical conditions.
5. There is no protection for international sharing of DNA profiles.

Q.20) Recently ministry of corporate affairs (MCA) has deregistered about 50,000 companies.
Highlight the threats posed by illegal shell companies and steps taken by government to deal with
them. (GS-3)

Answer: Shell companies include multiple layers of companies that have been created for the purpose of
diverting money or for money laundering.

Threats posed by illegal shell companies:


1. These companies are sources or destinations of money laundering. Thus, they can convert money
gained from illegal activity into legal money.
Created with by ForumIAS.com – The Knowledge Network for Civil Services. Visit http//forumias.com New!
http://forumiasacademy – Prelims & Mains Test Series

ForumIAS
MAINS MARATHON COMPILATION August, 2018

2. Often these companies misuse their corporate structure for various illegal purposes.
3. Sometimes shell companies are created for the purpose of evading tax. They affect the tax
revenues.

Steps taken so far:


1. Ministry of Corporate Affairs(MCA) and Central Board of Direct Taxes (CBDT) signed a
Memorandum of Understanding (MoU) to facilitate the sharing of data and information with
each other on an automatic and regular basis.
2. SEBI directed stock exchanges to initiate action against 331 suspected shell companies and bar
them from trading.
3. MCA cancelled the registration of around 2 lakh defaulting companies and Ministry of Finance
directed banks to restrict operations of bank accounts of such companies by the directors of such
companies. MCA has identified directors for disqualification under Companies Act, 2013.
4. A database of shell companies was compiled by the SFIO.
5. The constitution of the Special Investigation Team on Black Money.
6. Enactment of The Black Money (Undisclosed Foreign Income and Assets) and Imposition of Tax
Act, 2015, Income Declaration Scheme, 2016, Benami Transactions (Prohibition) Amendment Act,
2016 are measures taken from the legal front.
7. Demonetisation scheme

Steps that can be taken:


1. Clean up the corporate registry from time to time.
2. Create a transparent and compliant corporate ecosystem.
3. Promote ease of doing business and simplify tax compliance for corporates.

Q.21) Instances of people losing their hard-earned money to Ponzi schemes keep coming to light.
What are Ponzi schemes? How these schemes run unhindered in India? In your opinion, what steps
should be taken to curb the menace of these schemes? (GS-3)

Answer: A Ponzi scheme is a fraudulent investing scam promising high rates of return with little risk to
investors. Schemes run by Sharda group have led to a major crisis in West Bengal.

How these schemes run despite many failures:


1. People are often lured by promises of quick gains.
2. Lack of financial literacy in rural areas makes people vulnerable to deceit.
3. There are no regulatory measures to prevent such scheme.
4. Poor financial inclusion in the country attracts the poor to such illegal sources of money(for
credit).

Steps to be taken:

1. The Banning of Unregulated Deposit Schemes Bill, 2018 aims to provide comprehensive legislation to
deal with illicit deposit schemes.
a) Complete prohibition of unregulated deposit taking activity.
b) Deterrent punishment for promoting or operating an unregulated deposit taking scheme.
Created with by ForumIAS.com – The Knowledge Network for Civil Services. Visit http//forumias.com New!
http://forumiasacademy – Prelims & Mains Test Series

ForumIAS
MAINS MARATHON COMPILATION August, 2018

c) Punishment for default in repayment to depositors.


d) Designation of a competent authority by State government to ensure repayment of deposits in the
event of default.
e) Attachment of assets of a defaulting establishment.
f) Designation of courts to oversee repayment of depositors and to try offences under the Act.
g) Listing of Regulated Deposit Schemes in the Bill. Central government can expand or prune the list.
h) It also bans deposit takers from promoting, operating, issuing advertisements or accepting deposits in
any Unregulated Deposit Scheme.

2. Increasing Financial Literacy in the citizens.


3. Stricter implementation of fines and punishments.
4. Expanding financial inclusion.

Q.22) Invasive species have contributed to 40% of all animal extinctions since the 17th century, discuss
the implications of the invasive species on the local environment and economy. What international
mechanisms are available to deal with them? (GS-3)

Answer: An invasive species can be any kind of living organism that is not native to an ecosystem and
causes harm. They can harm the environment, the economy, or even human health.
Implications on local environment:
1. Widespread loss of habitat
2. Some invaders can physically alter the habitat. Eg., change in the nature of trees that survive
depending on the new species
3. Invading species destroy endemic species diversity as, often, they do not have any predators.

Impact on the economy:


1. Invasive species can also impact human health. Sometimes, invasive animals can also be vectors
for disease.
2. As they impact the local habitat, they affect the livelihoods of people depending on local
resources life trees, ponds, fishes etc., In Tamil Nadu the Seemai karuvelam tree was blamed for
affecting groundwater resources.
3. The introduction and spread of an invasive species can have major trade implications. There is
the prospect of losing a competitive advantage in exports because unaffected countries will either
prohibit import of goods from affected countries or establish costly precautionary measures

Mechanisms to deal them:


1. IUCN has formulated guidelines for managing invasives specifically in islands. The mechanism
involves data collection, community engagement, policy measures and management plans.
2. The Convention on Biological Diversity (CBD) – Article 8(h) of the CBD states that parties shall
prevent the introduction of alien species which threaten ecosystems, habitats or species.
3. Agreement on the Application of Sanitary and Phytosanitary Measures – It provides an
international legal basis for all sanitary and phytosanitary measures that affect international
trade.

Created with by ForumIAS.com – The Knowledge Network for Civil Services. Visit http//forumias.com New!
http://forumiasacademy – Prelims & Mains Test Series

ForumIAS
MAINS MARATHON COMPILATION August, 2018

4. CITES – The aim of Convention on International Trade in Endangered Species is to ensure that
international trade of wild animals and plants does not threaten their survival.
5. Convention on Migratory Species of Wild Animals (CMS or Bonn Convention) – it aims to
conserve terrestrial, marine and avian migratory species.
6. Convention on Wetlands (Ramsar Convention) – COP 7 resolution addresses threats of invasive
species to wetland ecosystems.
7. International Convention for the Control and Management of Ship’s Ballast Water and Sediments
– Not yet in force. It provides guidance and strategies to minimize and eliminate the transfer of
harmful aquatic organisms and pathogens from ballast water and sediments.
8. International Plant Protection Convention (IPPC) – it aims to prevent introduction of pests of
plants and plant products in international trade.
9. United Nations Convention on the Law of the Sea (UNCLOS) – States are required to protect and
preserve the marine environment from intentional or unintentional introduction of alien species.
10. World Organisation for Animal Health (OIE) Agreement – to guarantee the safety of
international trade in animals and animal products and to control animal diseases and zoonoses
worldwide while avoiding unjustified sanitary barriers.

Q.23) Critically analyze the reasons for which union government has recently banned the import of
oxytocin and also discuss the legal provisions governing drugs ban in India. (GS-3)

Answer: Recently the Union Health Ministry imposed a ban on the retail sale and private manufacture of
oxytocin. Government also asked customs officials to step up vigilance against those likely to smuggle
oxytocin into India.

Why it was banned:


1. Misuse in the dairy and poultry industry – Because oxytocin stimulates lactation in cattle, dairy
farmers inject the drug indiscriminately to increase milk production. This has spawned several
unlicensed facilities that manufacture the drug for veterinary use. As it is also a growth hormone,
it is introduced into the poultry for quick fertility and increased production.
2. Harm to cattle farming – Overuse of the hormone oxytocin leads to infertility in dairy animals.
Activists also say that it causes hormonal imbalances and shortens the lives of milch animals and
makes them barren sooner. It has also been linked to mastitis, a painful inflammation of the
udder of the animals.
3. Impact on human beings – Milk consumers worry about exposure to diseases life Mastitis
through dairy products. There are also concerns that oxytocin can enter the food chain through
milk that can be harmful for human consumption(though it was not established).
4. To promote use of genuine product. Govt orders that the sole PSU producing it to supply it
directly to registered private and public hospitals.
5. There is large-scale clandestine manufacture and sale of the drug leading to its grave misuse,
which is harmful to animals and humans.

Created with by ForumIAS.com – The Knowledge Network for Civil Services. Visit http//forumias.com New!
http://forumiasacademy – Prelims & Mains Test Series

ForumIAS
MAINS MARATHON COMPILATION August, 2018

Provisions governing drug ban:


1. Drug Technical Advisory Board can recommend the prohibitions under section 10 of the drugs
and cosmetics act 1940. In case of Oxytocin, DTAB agreed to prohibit the import of oxytocin and
its formulations.
2. Under Schedule H of the Drugs and Cosmetics Rule, 1954, a drug can be distributed by
prescription and only by a registered medical practitioner.
3. Drugs and Cosmetics Act and Rules
4. Narcotic Drugs and Psychotropic Substances (NDPS) Act – finance ministry regulates certain
sections of the NDPS Act like categorisation of pharma drugs as narcotic substances.

Q.24) What are the reasons of urban flooding in India? Discuss the mechanism available to deal with
urban flooding and why it has proved to be ineffective in times of need? GS-3

Answer: Urban flooding is the inundation of land or property in more densely populated areas, caused
by rainfall overwhelming the capacity of drainage systems, such as storm sewers.

Reasons for urban flooding:


Leaving aside the natural causes like torrential rainfall and lack of natural water bodies in many urban
areas, man-made factors play a major role in urban floods. These are:
1. Urbanisation and increasing pressure on land. The urban areas are highly dense making them
more vulnerable to disasters like floods.
2. Land use changes life levelling, deforestation.
3. Settlements along flood plains
4. Poor infrastructure to dispose municipal solid wastes and for drainage of sewage water.
5. Urban heat islands and micro-climate changes in urban areas

Mechanisms to deal with urban floods:


1. The NDMA has the responsibility of laying down policies, plans and guidelines for DM and
coordinating their enforcement and implementation for ensuring timely and effective response to
disasters.
2. The DM Act 2005 has mandated the constitution of NDRF for the purpose of specialised response
to a threatening disaster situation or disaster.
3. All States and UTs will be required to train some personnel of their existing armed police
battalions in DM, as they are critical first responders.
4. Ministry of Water Resources and Central Water Commission have been the nodal ministry and
agency for dealing with Floods, focusing on riverine floods affecting large extents of rural areas.
5. NDMA decided to address urban flooding as a separate disaster de-linking it from Floods and
evolved separate guidelines. Taking into consideration the fact that stormwater drainage system
is one of the important components of UFDM, it is allotted to Ministry of Urban Development
(MoUD).
6. IMD is the custodian of rainfall data in India.

Created with by ForumIAS.com – The Knowledge Network for Civil Services. Visit http//forumias.com New!
http://forumiasacademy – Prelims & Mains Test Series

ForumIAS
MAINS MARATHON COMPILATION August, 2018

Why it is not effective:


1. Most of the times, urban infrastructure does not include risk planning. Many local bodies fail to
meet the infrastructure requirements, leave aside the disaster preparedness.
2. The communication networks often fail due to lack of sufficient awareness and preparedness
among the officials from top to the local levels.
3. The NDRF teams are not sufficiently manned to meet the emergencies across the country. Most of
the state governments failed to develop their own standalone forces as instructed by the DM Act.
4. The IMD data is at a more broader level and not specific to localities. This creates challenges in
assessing the risks at a particular location at a time.
5. Rapid urbanisation has made the urban planning limited and inefficient in response.

Q.25) What is Biofuel. Discuss its applications, benefits and challenges in its application. (GS-3)

Answer: A biofuel is a fuel that is produced through contemporary biological processes, such as
agriculture and anaerobic digestion, rather than a fuel produced by geological processes such as those
involved in the formation of fossil fuels, such as coal and petroleum, from prehistoric biological matter.

Ex: Ethanol, Syngas, Biodiesel

Biofuels can be derived directly from plants (i.e. energy crops), or indirectly from agricultural,
commercial, domestic, and/or industrial wastes.

Biofuels are categorised based on the type of feedstock (the input material) used to produce them.

First generation – produced from food crops. For ethanol, feedstocks include sugar cane, corn, maize,
etc. For biodiesel, feedstocks are naturally occurring vegetable oils such as soybean and canola[

Second generation – produced from cellulosic material such as wood, grasses, and inedible parts of
plants. This material is more difficult to break down through fermentation and therefore requires pre-
treatment before it can be processed

Third generation – produced using the lipid production from algae. algaculture unlike crop-based
biofuels, does not entail a decrease in food production, since it requires neither farmland nor fresh water.

Fourth generation – Similar to third-generation biofuels, fourth-generation biofuels are made using non-
arable land. However, unlike third-generation biofuels, they do not require the destruction of biomass

Applications

Replacements for transportation fuels like petroleum, diesel and jet fuel Biofuel can provide hydrogen,
clean up oil, work as cooking oil Biofuels can work as an alternative to replacing energy needs life central
home heating. Biofuel can be used to generate power in backup systems Biofuel can replace the toxic
products that are designed to remove paint and adhesives

Created with by ForumIAS.com – The Knowledge Network for Civil Services. Visit http//forumias.com New!
http://forumiasacademy – Prelims & Mains Test Series

ForumIAS
MAINS MARATHON COMPILATION August, 2018

Benefits

Biofuels are in theory carbon-neutral – the carbon dioxide that is absorbed by the plants is equal to the
carbon dioxide that is released when the fuel is burned Produce less greenhouse gases overall than fossil
fuels when they are burned.
Alternative sources to fossil fuels – allowing for greater fuel security for countries with little or no oil
reserves of their own.
May not produce any particulates such as soot and other fine particles.
Cost benefit of using them is much higher. Biofuels are adaptable to current engine designs and perform
very well in most conditions. This keeps the engine running for longer, requires less maintenance and
brings down overall pollution check costs.

Since most of the sources like manure, corn, switchgrass, soyabeans, waste from crops and plants are
renewable and are not likely to run out any time soon, making the use of biofuels efficient in nature.
These crops can be replanted again and again.

Challenges

High Cost of Production – Even with all the benefits associated with biofuels, they are quite expensive to
produce in the current market. Biofuels are similar to fossil fuels in that biofuels contribute to air
pollution.

Large quantities of water are required to irrigate the biofuel crops and it may impose strain on local and
regional water resources, if not managed wisely.

Risk of diverting farmland or crops for biofuels production to the detriment of the food supply It might
be economically attractive for farmers but growing same crop every year may deprive the soil of
nutrients Biofuels are produced from crops and these crops need fertilizers to grow better. The downside
of using fertilizers is that they can have harmful effects on surrounding environment

Q.26) Discuss the causes and implications of falling Rupee value at present in India. GS-3

Answer: Rupee crashed to a historic low of above 70 value recently.

Causes:
1. Turkish crisis – the ongoing fall in the value of Turkish lira has resulted in the fall of many global
currencies. This is due to the defaults of
2. Rising oil prices – India imports more than 80 percent of its crude-oil needs.
3. US monetary tightening – moves by US Federal Reserve to lift interest rates and end its
“quantitative easing”. Thus it reverses the flow of relatively cheap dollar-denominated loans.
4. Rising protectionism in the form of tariffs by US on steel and aluminum imports and possible US
trading restrictions on countries trading with Iran.
5. Investors – Foreign portfolio investor (FPI) outflows due to the need to gain from a strengthening
dollar.

Created with by ForumIAS.com – The Knowledge Network for Civil Services. Visit http//forumias.com New!
http://forumiasacademy – Prelims & Mains Test Series

ForumIAS
MAINS MARATHON COMPILATION August, 2018

Implications:
1. Imports – a weaker rupee will not favour all sections of Indian industries as those which rely on
imported raw materials, component parts and machinery will face increases in their
manufacturing costs in rupee terms.
2. Exports – weaker rupee is seen as a favourable factor for exports as it will make the country’s
products competitively cheaper in the world market.
3. Oil imports will get costlier. This could lead to inflation.
4. Inflation – a weaker rupee could lead to higher inflation under conditions of increases in import
bills of crude-oil, commodities, electronic items and engineering equipment.
5. The current level of forex reserves about $402 billion would not cover import costs for a year.

Q.27) What are the causes behind disputes over land acquisitions in India. Would Land acquisition
(second amendment) bill prove to be effective in dealing with these disputes? provide valid
arguments in favour of your views(GS-3)

Answer: Land Acquisition has been a contentious issue in Indian development scene.
Reasons for disputes over land acquisition:
1. Most of the times, the displaced are not made partners of development.
2. The compensation is often in the form of cash or land not as fertile as the acquired land. It cannot
fully compensate for the losses of the displaced.
3. Rehabilitation does not consider the social and cultural costs of development. Particularly for
tribals attached to the land for their way of life, displacing from their native land is equal to
death.
4. The calculation of compensation is not done at market cost. Also, there is inadequate
representation of locals in determining the rightful compensation.
5. The people often facing the displacement are tribals or villagers. The development benefits life
dams or infrastructure highways seldom suit their needs. This brings the case of prioritizing
one’s needs over other.
6. Displacement for projects life hydropower dams not only has economic cost, but also bring in
huge ecological damage to the locals. They pose the risk of submergence of villages.
7. The Impact Assessment process is not representative and highly bureaucratic. Most of the times,
vested interests play a dominating influence over the outcomes of the process.

The Right to Fair Compensation and Transparency in Land Acquisition, Rehabilitation and Resettlement
(Second Amendment) Bill, 2015 amends the Act passed in 2013. Provisions of the bill:
1. It enables the government to exempt five categories of projects from the requirements of: (i) social
impact assessment, (ii) restrictions on acquisition of multi-cropped land, and (iii) consent for
private projects and public private partnerships (PPPs) projects.
2. The five categories of projects are: (i) defence, (ii) rural infrastructure, (iii) affordable housing, (iv)
industrial corridors, and (v) infrastructure including PPPs where government owns the land.
3. The Act would apply retrospectively, if an award had been made five years earlier and
compensation had not been paid or possession not taken.
Created with by ForumIAS.com – The Knowledge Network for Civil Services. Visit http//forumias.com New!
http://forumiasacademy – Prelims & Mains Test Series

ForumIAS
MAINS MARATHON COMPILATION August, 2018

The bill speeds up the process of :and Acquisition as a process. It thus makes the development of
infrastructure across India speedy. This is in line with the government’s development agenda of
Transforming India. But it fails to cater to the demands of the people involved in it.

Problems with the bill:


1. The five types of projects being exempt from the provisions of social impact assessment,
restrictions in case of multi-cropped land and consent are broad and may cover many public
purpose projects.
2. The Act requires consent of 70% of landholders for PPP projects, and 80% for private projects.
Acquisition, is different from purchase and implies that landowners are unwilling to part with
the land. Requiring consent from them may be impractical. Also, it is not clear why the consent
requirement depends on who owns the project.
3. It gives preference to speedy acquisition over building consensus in the citizens.

Q.28) Highlight the need for National security reforms and restructuring in India and discuss the steps
taken by government towards this goal. GS-3

Answer: Need for national security reforms and restructuring:


1. Increasing threats from the northern and western border. This is understood from the Doklam
crisis and cross border infiltration and attacks at Pathankot and Uri.
2. Rising cyber threats and need for a cyber warfare in times of contingency.
3. Failure to build a robust defence-industrial base.
4. Lack of long term strategic planning to build military capabilities.
5. India’s growing geopolitical presence and super power ambitions need a different set of defence
infrastructure.

Steps taken so far:


1. The new Defence Planning Committee(DPC) was created to create a coherent plan for national
defence which integrates all national forces and addresses all threats.
2. The new Defence Procurement Policy gives a boost to domestic production of arms and enables
technology transfer.
3. The reform initiatives in army were recommended by a committee headed by Shekatkar. They
include:
a) Better utilization of Supply and Transportation echelons and Animal Transport Units.
b) Closure of Military Farms and Army Postal Establishments in peace locations.
c) Enhancement in standards for recruitment of clerical staff and drivers in the Army.
d) Improving the efficiency of the National Cadet Corps.
e) Redeployment of Ordnance echelons to include Vehicle Depots, Ordnance Depots and Central
Ordnance Depots apart from streamlining inventory control mechanisms.
f) Optimization of Signals Establishments to include Radio Monitoring Companies, Corps Air
Support Signal Regiments, Air Formation Signal Regiments, Composite Signal Regiments and
merger of Corps Operating and Engineering Signal Regiments.

Created with by ForumIAS.com – The Knowledge Network for Civil Services. Visit http//forumias.com New!
http://forumiasacademy – Prelims & Mains Test Series

ForumIAS
MAINS MARATHON COMPILATION August, 2018

Q.29) Reliable Internet connectivity has the potential to bridge the rural-urban economic divide and
revolutionise healthcare and education. Explain. (GS-3)

Answer: India’s internet consumption largely remains an urban phenomenon. According to the report
Internet in India 2017, while internet penetration in urban India was at 64.84%, it was 20.26% in rural
India.

How internet can bridge rural-urban economic divide:


1. Internet connectivity provides more self-employment opportunities for people living in rural
areas.
2. Agriculture
a) It can revolutionise the price realisation for farmers and expand their market.
b) New technologies enable small farmers to shift from input-intensive to knowledge-intensive
agriculture.
c) Precision agriculture can improve the timeliness of planting, secure the best market prices
through market information and e-market reforms.
d) Provide fertiliser subsidies via direct bank transfers that eliminate or reduce the cost of
financial intermediaries
e) Improve agricultural extension.
f) Digitised land registration, mobile phones and ‘Uberised’ tractor services all are contributing to
improved farm management.
1. Rapid skilling of village youth helps in tapping the demographic dividend.

How internet can revolutionise healthcare and education:


Education :
1. It can bring tele education and address literacy gap between rural and urban areas. Most of the
success stories in education sector come from urban areas. Internet gives access to the vast world
of knowledge and empowers rural youth and children to learn.
2. The cost of infrastructure can be substantially reduced. E-schools can achieve learning outcomes
with lesser buildings and less no of teachers/ mentors.
3. Modern methods of teaching through AV aids can be utilised by teachers in primary schools with
internet connectivity. This will address the problem of learning outcomes and dropouts as
education becomes easy.
Health:
1. It bridges the gap in providing primary healthcare to villagers. Most of the PHCs are not
sufficiently manned and do not even meet the necessary standards. With e-health solutions,
primary healthcare can be easily provided and the cost of healthcare reduces substantially.
2. Management of public health can become easy. The prediction of diseases, monitoring of health
of individuals can be simplified if the rural health records are connected.

Created with by ForumIAS.com – The Knowledge Network for Civil Services. Visit http//forumias.com New!
http://forumiasacademy – Prelims & Mains Test Series

ForumIAS
MAINS MARATHON COMPILATION August, 2018

Q.30) Critically assess the need of military reform in India, giving special importance to
recommendations of Naresh Chandra Committee. (GS-3)

Answer: Need for military reform in India:


1. Boundary disputes with Pakistan and China – China has rapidly modernized its armed forces
and has emerged as a major arms developer and exporter. This poses a challenge to India.
2. Successive committee reports have pointed to the need for a unified command structure in India
due to the lack of coordination between the three armed forces.
3. Warfare in the cyber and space domains affect all the arms. Fast moving digital battlefield
requires flatter decision-making structures as compared to multiple verticals.
4. Proposal about a tri-service chief, first mooted by the Arun Singh Committee in 1990, is yet to be
implemented.
5. 72% of budget goes into cost of personnel, 17% for maintenance of equipment and just 11% for
new equipment.
6. Indian military remains structured in the same way that it was thirty or even forty years ago.

Naresh Chandra committee recommendations:


1. Creation of a new post of Intelligence Advisor to assist the NSA and the National Intelligence
Board on matters relating to coordination in the functioning of intelligence committee
2. Amendment to Prevention of Corruption Act to reassure honest officers, who take important
decisions about defence equipment acquisition, so that they are not harassed for errors of
judgement or decision taken in good faith.
3. A permanent Chairman of the Chiefs of Staff Committee
4. Expediting the creation of new instruments for counter-terrorism, such as the National
Intelligence Grid and National Counter Terrorism Centre.
5. Deputation of officers from services up to director’s level in Ministry of Defense
6. Measures to augment the flow of foreign language experts into the intelligence and security
agencies, which face a severe shortage of trained linguists
7. Promotion of synergy in civil-military functioning to ensure integration. To begin with, the
deputation of armed services officers up to director level in the Ministry of Defence should be
considered.
8. Early establishment of a National Defence University (NDU) and the creation of a separate think-
tank on internal security.

Q.31) Discuss the factors behind India’s ailing indigenous defense manufacturing. How new Strategic
Partnership guidelines would be able to transform defense manufacturing in India? (GS-3)

Answer: Challenges in indigenous defence manufacturing:


1. Subhash Bhamre committee says that “Make in India” initiative for the defence sector continues
to languish due to procedural delays.
2. The FDI in defence manufacturing is still at 49%. This does not incentivise foreign investments
into India.

Created with by ForumIAS.com – The Knowledge Network for Civil Services. Visit http//forumias.com New!
http://forumiasacademy – Prelims & Mains Test Series

ForumIAS
MAINS MARATHON COMPILATION August, 2018

3. The ‘offset policy’ is not effective most of the times. Many important contracts like the Multi-role
Tanker Transport Aircraft (mid-air refueller) and the M-777 howitzer have faced considerable
delays due to the offsets problem.
4. The technology transfers under the defence deals do not favour India. Most of the time, the vital
components of technology are not transferred.
5. Lack of investment in defence production. Most of the defence budget(76%) goes into personnel
only. Just 17% of it is spent on acquiring new technologies.
6. The defence production at present is concentrated in the govt-run OFBs which are inefficient and
lack updated technologies.
7. The necessary skills and research environment for the defence production is lacking in India.

Strategic Partnership guidelines: this is intended to institutionalise a mechanism to encourage broader


participation of the private sector in the manufacture of defence platforms and equipment. The following
four segments have been identified for acquisition under Strategic Partnership (SP) route:
a) Fighter Aircraft
b) Helicopters
c) Submarines
d) Armoured Fighting Vehicles (AFVs) / Main Battle Tanks (MBTs).

Efficacy:
1. This brings in private sector participation into defence production, which has been dominated by
public sector units so far. This will bring in competition into the sector.
2. The much needed investments into the sector will come.
3. This encourages cutting edge research in new technologies as private partners aim for profits.
4. More MSMEs will enter the sector as the whole defence industry along with its ancillary
suppliers will grow.

Created with by ForumIAS.com – The Knowledge Network for Civil Services. Visit http//forumias.com New!
http://forumiasacademy – Prelims & Mains Test Series

ForumIAS
MAINS MARATHON COMPILATION August, 2018

General Studies – 4

Q.1) Is morality important to lead a happy life? Support your answer with examples. GS 4

Morality can be defined as principles concerning the distinction between right and wrong or good and
bad behavior. In our day to day lives we often face situations where we have to choose between options
and moral character of a person plays big part in it. Morality is definitely a very important value to lead a
happy life. A person with high integrity and moral character will choose the right and we know that the
path of truth will lead to happiness in the long term even at the cost of short term sufferings.

In his childhood. Gandhi Ji once stole some money from his father’s pocket. He knew that his father
would never come to know about it. However, showing high moral character, he confessed his mistake.
The rest is history and we know Mohan Das became father of the nations.

There are numerous examples in our daily life where a moral life can lead to happiness. Some of them are
explained below:
1. Corrupt vs. Honest Officers – Former CAG Mr. Vinod Rai unearthed big scams and was at the
receiving end of the then government. However, now he is respected by all. At the same time, a
dishonest officer can accumulate huge wealth but will always live in fear
2. Tax Evasion – People who pay taxes honestly are living happily without any fear versus tax
evading people are always in fear of raid or penal action
3. Every day scenarios such as jumping red light, bribing traffic police, cheating in exams test out
morality and people who are at the right side are always happy in the long term
There are many other ethical values in addition to morality such as honestly, integrity, truthfulness which
are required to lead a happy and satisfying life. It is the responsibility of a society to teach them to our
childrens so that overall we became a happy world.

Q.2) “Emotional intelligence is your ability to recognize and understand emotions in yourself and others,
and your ability to use this awareness to manage your behavior and relationships.: Analyze. (GS4)

Emotional intelligence:- Four main skills of emotional intelligence are:-


• Self-awareness– our ability to perceive our emotions and understand our tendencies to act in
certain ways in given situations
• Social awareness– our ability to understand the emotions of other people, what they are thinking
and feeling
• Self-management– our ability to use awareness of our emotions to stay flexible and direct our
behavior positively and constructively
• Relationship management– our ability to use our awareness of our own emotions and those of
others to manage interactions successfully

Created with by ForumIAS.com – The Knowledge Network for Civil Services. Visit http//forumias.com New!
http://forumiasacademy – Prelims & Mains Test Series

ForumIAS
MAINS MARATHON COMPILATION August, 2018

Importance of emotional intelligence:


1.At work place:
• An employee with low emotional intelligence can negatively impact a workplace and their team
members leading to poor morale.
• Examples of low emotionally intelligent behaviour includes; not being able to take critical
feedback, laying blame on other staff, passive-aggressive comments.
In management, those with low emotional intelligence exhibit the same traits, but can also be
leaders who do not listen to the recommendations of the staff that they manage and become out of touch
with those that they lead.

Self-Awareness, Self-Regulation and Self-Motivation


• Employees who possess high levels of emotional intelligence are much more self-aware.
• In the workplace this translates to an employee who understands their own strengths and their
own weaknesses in addition to how their actions could affect their team members.
• Self-aware employees are also better equipped to handle constructive criticism and learn from
their mistakes.
• An employee with high emotional intelligence can also reveal and control their own emotions to
team members
• Emotionally intelligent people are also self-motivated, but they are not motivated by money or a
title alone

Empathy and Interpersonal Skills


• An empathetic employee is an employee who has compassion and understands human nature.

Q.3) Citizen’s Charter helps maintaining a healthy work culture. Do you agree? Explain with
Examples. (GS-4)

Answer: Citizen’s charters’ main objective is to improve the quality of public services. This is done by
letting people know the mandate of the concerned Organisation, how one can get in touch with its
officials, what to expect by way of services and how to seek a remedy if something goes wrong.

How they help maintain healthy work culture:


1. They maintain a clear communication between various employees because they are together
answerable to the people.
2. It enables a clear accountability mechanism for each employee and thus reduces the scope for
subjectivity in assessing their performance. For example, an employee who caters to many
citizens’ requests can be clearly identified due to clearly specified duties.
3. It aligns the personal goals of each employee to the organisations goals and thus creates a spirit of
service for the organisation.
4. Citizen charters streamline the processes of the organisation to reduce time lag in service
delivery. Hence, employees do not find it burdensome to fulfil their mandated duties. For

Created with by ForumIAS.com – The Knowledge Network for Civil Services. Visit http//forumias.com New!
http://forumiasacademy – Prelims & Mains Test Series

ForumIAS
MAINS MARATHON COMPILATION August, 2018

example, the current physical filing system in time taking and cumbersome. It does not
incentivise the employee to speed up his work.
5. It centers the administration around citizen and makes employees more citizen-friendly.
6. It improves the efficiency of work by putting to use the available resources wisely.
7. It encourages fairness of treatment and reduces scope for nepotism, corruption at workplace.

Q.4) Differentiate between dilemma and ethical dilemma with suitable examples. How does
conscience act as a source of ethical guidance? (GS-4)
Answer: A dilemma can be regarding any decision man takes in his life. It can be over which dress to
choose or which ice cream to eat. Ethical dilemma is more specific. It is a dilemma about which is the
“right” way to act. Whether to reach office on time or help a victim in accident is an ethical dilemma as,
here, both the choices seem right.
In such cases conscience acts as a clear source for guiding the ethical action of individual. This is how:
1. Conscience tells a man what is right and wrong. Though it is considered wrong by others or
rules, it guides him.
2. A man feels pleasure when he does something good. He suffers pain on doing bad. Conscience
compels a man to pursue pleasure and discard pain. Thus it guides him to act ethically.
3. Conscience is formed from the learnings, experiences and beliefs of an individual. Even for a bad
person, his conscience guides him to do correct as that is what he has learnt over time.
4. Immanuel Kant expressed the idea of the critical conscience which was rather like a court of law
in our minds where the prosecutors or conscience excuse or accuse thoughts and actions.

Q.5) What do you understand by corporate social responsibility? What relationship does corporate
governance share with corporate social responsibility? (GS-4)

Answer: Corporate social responsibility is a business approach to contributes to sustainable development


by delivering economic, social and environmental benefits for all stakeholders.
IT is generally understood that following the principles of corporate governance will eventually lead to
CSR. The Companies Act prescribes a mandatory CSR spend of 2% of average net profits for companies.
Relation between CG and CSR:

Similarities:
1. CG is usually incorporated in the company’s governance mechanisms.
2. Both these concepts focus on the ethical aspects of doing business.
3. They lead to better image of the company and thus affects its performance positively.

Differences:
1. Fulfilling CSR norms but faulting on Corporate Governance is usually seen. Here, CSR is used as
a marketing strategy than genuine intention in helping all round development.
2. CG is related to profit maximisation while CSR is about sharing the profits for others’ welfare.
3. The officials of an organisation who are hired with due technical qualifications may not fulfil its
CSR mandates.
Created with by ForumIAS.com – The Knowledge Network for Civil Services. Visit http//forumias.com New!
http://forumiasacademy – Prelims & Mains Test Series

ForumIAS
MAINS MARATHON COMPILATION August, 2018

It has to be remembered that CG may not eventually lead to CSR. If a company visualises its long term
growth in terms of allround welfare, it may incorporate CSR principles into its CG strategy.

Q.6) What do you understand by “entrepreneurial governments”? Do you think civil servants need to
develop entrepreneurial behavior? (GS-4)

Answer: A government which takes an entrepreneurial approach in handling its responsibilities is called
entrepreneurial government. Just life free market and competition benefits the public at large, this kind of
government is expected to serve the citizens better.

Principles of entrepreneurial government:


1. It defines missions in terms of objectives.
2. The objectives are realistic.
3. It embraces change.
4. It is market-oriented.
5. Decentralisation
6. Customer driven
7. Community owned
Need for entrepreneurial approach:
1. It brings in efficiency in implementing government programs and thus cuts down government
budgets.
2. As it is more citizen-friendly, it gives preference to local needs rather than imposing programs in
a top-down approach.
3. Government sector’s performance is poor when compared to the private sector in service
delivery. These new principles can improve the quality of Public Service Delivery.
Problems with the approach:
1. It may lead to excessive privatisation, which is harmful in essential sectors life health and
education.
2. Welfare attitude of government is required in poor and developing countries. Business approach
may not always help.
3. Efficiency is needed, but it cant be given more priority over equity and effectiveness. For
example, though many schools are running successfully under private sector the question
whether they are accessible to all needs to be answered.

Created with by ForumIAS.com – The Knowledge Network for Civil Services. Visit http//forumias.com New!
http://forumiasacademy – Prelims & Mains Test Series

ForumIAS

You might also like